Maternity and Women's Health Final

Réussis tes devoirs et examens dès maintenant avec Quizwiz!

Which of the following nursing actions are directed at promoting bonding? (Select all that apply.) Select one or more: a. Providing opportunity for parents to hold their newborn as soon as possible following the birth. b. Providing opportunities for the couple to talk about their birth experience and about becoming parents. c. Promoting rest and comfort by keeping the newborn in the nursery at night. d. Providing positive comments to parents regarding their interactions with their newborn.

A & D a. Providing opportunity for parents to hold their newborn as soon as possible following the birth. CorrectParent bonding can be delayed by prolonged periods of separation from their child. The other three actions support parent bonding with their newborn. d. Providing positive comments to parents regarding their interactions with their newborn. CorrectParent bonding can be delayed by prolonged periods of separation from their child. The other three actions support parent bonding with their newborn.

18 year old Ellen has a positive pregnancy test and cries when she sees the result. Per her LMP, the nurse determines she is approx 7 weeks pregnant. She asks the nurse what should she do. Which of the following is the nurse's best initial response? Select one: a. Let's discuss your three options. b. Here is a list of clinics that provide abortions. My sister liked this one best. c. You need to decide what obstetrician you want to see for prenatal care. d. I think adoption would be your best choice at this time.

A

A couple who has sought infertility counseling has been told that the man's sperm count is very low. The nurse advises the couple that spermatogenesis is impaired when which of the following occur? Select one: a. The testes are overheated. b. The vas deferens is ligated. c. The prostate gland is enlarged. d. alcohol use

A

A nurse caring for a family during a loss might notice that a family member is experiencing survivor guilt. Which family member is most likely to exhibit this guilt? Select one: a. Grandparents b. Siblings c. Mother d. Father

A

In caring for an immediate postpartum client, the nurse notes petechiae and oozing from her intravenous (IV) site. The client would be closely monitored for which clotting disorder? Select one: a. Disseminated Intravascular Coagulation (DIC) b. Hemorrhage c. Amniotic fluid embolism (AFE) d. HELLP syndrome

A

Preconception counseling is critical in the safe management of diabetic pregnancies. Which complication is commonly associated with poor glycemic control before and during early pregnancy? Select one: a. Congenital anomalies in the fetus b. Hydramnios c. Hyperemesis gravidarum d. Frequent episodes of maternal hypoglycemia

A

The time from fertilization to implantation is _____ days. Select one: a. 6-7 b. 3-4 c. 7-8 d. 1-2

A

Which component of the sensory system is the least mature at birth? Select one: a. Vision b. Taste c. Hearing d. Smell

A

Which diagnostic test is used to confirm a suspected diagnosis of breast cancer? Select one: a. Needle-localization biopsy b. Ultrasound c. Magnetic resonance imaging (MRI) d. Mammogram

A

Which health care service represents a primary level of prevention? Select one: a. Immunizations b. Pap smear test c. Blood pressure screening d. Home care for high-risk pregnancies

A

Which is true regarding breast cyst? Select one: a. It can be tender or not b. It is usually fixed and hard c. Removal is the best treatment d. It requires surgery for diagnosis

A

Which nursing intervention should be immediately performed after the forceps-assisted birth of an infant? Select one: a. Assessing the infant for signs of trauma b. Measuring the circumference of the infant's head c. Administering prophylactic antibiotic agents to the infant d. Applying a cold pack to the infant's scalp

A

Which of the following STDs poses the least danger for the fetus of a mother who has the infection? Select one: a. trichomonas vaginalis b. gonorrhea c. herpes simplex virus d. syphilis

A

Which of the following antepartum tests is (are) used to evaluate a Maternal Serum Screening test that is negative for increased risk? Select one: a. Further followup test not necessary b. CVS c. Biophysical profile d. Amniocentesis Feedback

A

Tobacco use during pregnancy is associated with adverse effects on the unborn infant such as intrauterine growth restriction, preterm births, and respiratory problems. By race, which has the highest percentages of smokers? Select one: a. American Indian and Alaskan Natives b. Asian or Pacific Islanders c. Non-Hispanic blacks d. Non-Hispanic whites

A See page 6 The correct answer is: American Indian and Alaskan Natives

Developing a realistic birth plan with the pregnant woman regarding her care is important for the nurse. How would the nurse explain the major advantage of nonpharmacologic pain management? Select one: a. No side effects or risks to the fetus are involved. b. Greater and more complete pain relief is possible. c. The woman will remain fully alert at all times. d. Labor will likely be more rapid.

A plus less risk to the mother

A couple is undergoing an infertility workup. The semen analysis indicates a decreased number of sperm and immature sperm. Which of the following factors can have a potential effect on sperm maturity? (Select all that apply.) Select one or more: a. The man rides a bike to and from work each day. b. The man takes a calcium channel blocker for the treatment of hypertension c. The man drinks 6 cups of coffee a day. d. The man was treated for prostatitis 12 months ago and has been symptom free since treatment.

A B The daily riding of a bike can be the cause of prolonged heat exposure to the testicles. Prolonged heat exposure is a gonadotoxin. A number of medications, such as calcium channel blockers, can have an effect on sperm production. Coffee has not been associated with low sperm counts. Prostatitis or other infections within the last 3 months may have an effect on

A thorough abuse assessment screen should be completed on all female clients. This screen should include which components? (select all that apply) Select one or more: a. Asking the client if she has been forced to perform sexual acts b. Asking the client if she has ever been slapped, kicked, punched, or physically hurt by her partner c. Asking the client if she is afraid of her partner d. Asking the client what she did wrong to elicit the abuse

A, B, C See Page 106 of Textbook

Which of the following are primary risk factors for subinvolution of the uterus? (Select all that apply.) Select one or more: a. Fibroids b. Retained placental tissue c. Metritis d. Urinary tract infection

A, B, C a. Fibroids Uterine fibroids can interfere with involution. Retained placental tissue does not allow the uterus to remain contracted. Infection in the uterus is a risk factor for subinvolution. UTI does not interfere with involution of the uterus. b. Retained placental tissue Uterine fibroids can interfere with involution. Retained placental tissue does not allow the uterus to remain contracted. Infection in the uterus is a risk factor for subinvolution. UTI does not interfere with involution of the uterus. c. Metritis Uterine fibroids can interfere with involution. Retained placental tissue does not allow the uterus to remain contracted. Infection in the uterus is a risk factor for subinvolution. UTI does not interfere with involution of the uterus. worsening preeclampsia is

Which of the following actions can decrease the risk for a postpartum infection? (Select all that apply.) Select one or more: a. Diet high in protein and vitamin C b. Increased fluid intake c. Ambulating within a few hours after delivery d. Good hand washing techniques by staff and patients

A, B, C & D Protein and vitamin C assist with tissue healing. Rehydrating a woman after delivery can assist with decreasing risk for infections. Early ambulation decreases risk for infection by promoting uterine drainage. Hand washing by staff and patients has been shown to be the number one measure in the transmission of infection.

A first-time mother informs her nurse that she is concerned about infant abduction. The nurse should explain to the parents which of the following? (Select all that apply.) Select one or more: a. Do not allow a person without proper unit specific hospital ID to take their baby. b. Encourage parents to accompany any person who removes their infant from the hospital room c. Instruct parents not to leave their newborn unattended at any time d. Inform parents that ID bands with matching identification numbers are placed on the parents and infant at birth to ensure identification of the correct infant with the correct parents

A, B, C, D

An 18-year-old woman at 23 weeks' gestation tells the nurse that she has fainted two times. The nurse teaches about the warning signs that often precede syncope so that she can sit or lie down to prevent personal injury. Warning signs include (select all that apply): Select one or more: a. Sweating b. Nausea c. Chills d. Lightheadedness

A, B, D Sweating is a warning sign that often precedes syncope. Syncope (a transient loss of consciousness and postural tone with spontaneous recovery) during pregnancy is frequently attributed to orthostatic hypotension or inferior vena cava compression by the gravid uterus. Nausea and yawning are warning signs that often precede syncope. Lightheadedness, sweating, nausea, yawning, and feelings of warmth are warning signs that often precede syncope. Chills are not a warning sign that often precede syncope.

An infant at 26 weeks of gestation arrives intubated from the delivery room. The nurse weighs the infant, places him under the radiant warmer, and attaches him to the ventilator at the prescribed settings. A pulse oximeter and cardiorespiratory monitor are placed. The pulse oximeter is recording oxygen saturations of 80%. The prescribed saturations are 92%. What are the nurse's most appropriate actions at this time? Select one or more: a. increase oxygen b. listen to breath sounds c. notify parents that their infant is not doing well d. ensure patency of the endotracheal tube e. complete the admission process and thorough assessment f. notify physician g. continue to observe until saturations are 75%

A, B, D, F

For which of the following reproductive cancers is high risk HPV infection thought to be primarily responsible? (Select all that apply). Select one or more: a. Vulvar cancer b. Endometrial cancer c. Vaginal cancer d. Ovarian cancer e. Cervical cancer

A, C, E

One of the most important components of the physical assessment of the pregnant client is the determination of BP. Consistency in measurement techniques must be maintained to ensure that the nuances in the variations of the BP readings are not the result of provider error. Which techniques are important in obtaining accurate BP readings? (Select all that apply.) Select one or more: a. The same arm should be used for every reading b. The cuff should cover a minimum of 60% of the upper arm. c. The client should be seated d. An electronic BP device should be used. e. The client's arm should be placed at the level of the heart.

A, C, E

Which client(s) should the nurse report to the health care provider? (Select all that apply). Select one or more: a. One 2-3mm sized mobile, nontender mass in lower outer quadrant of left breast. b. Left breast slightly smaller than right breast c. Eversion (elevation) of both nipples d. Small dimple located in the upper outer quadrant of the right breast

A, D

Which of the following are true regarding interpreter services? Select all that apply. Select one or more: a. Direct questions to the patient, not the interpreter. b. It is not necessary to worry about different dialects of a patient's native language when using interpreters. c. Never have a patient's child serve as an interpreter. d. Create environment of respect and privacy.

A, D Your answer is correct. The correct answer is: Direct questions to the patient, not the interpreter., Create environment of respect and privacy.

A healthy, full-term baby boy is scheduled for a circumcision. Nursing actions prior to the procedure include which of the following? (Select all that apply.) Select one or more: a. Obtain written consent from the parents b. Administer acetaminophen PO 1 hour before procedure per provider order. c. Feed the neonate glucose water 30 minutes before the procedure. d. Verify that the neonate has voided

A,B, and D- Obtain written consent from the parents, Administer acetaminophen PO 1 hour before procedure per provider order., Verify that the neonate has voided, neonate should not eat 2-3 hours prior to the procedure to avoid risk of vomiting and aspiration

The let-down reflex occurs in response to the release of oxytocin. Which of the following can stimulate the release of oxytocin? (Select all that apply.) Select one or more: a. Emotional response to thinking about her baby b. Infant suckling c. Emotional response to hearing an infant crying d. Sexual activity

A,B,C, and D-Emotional response to thinking about her baby, Infant suckling, Emotional response to hearing an infant crying, Sexual activity

Pain should be regularly assessed in all newborns. If the infant is displaying physiologic or behavioral cues that indicate pain, then measures should be taken to manage the pain. Which interventions are examples of nonpharmacologic pain management techniques? (Select all that apply.) Select one or more: a. Nonnutritive sucking b. Skin-to-skin contact with the mother c. Sucrose d. Acetaminophen e. Swaddling

A,B,C, and E- Sucrose, Skin-to-skin contact with the mother , Swaddling, Nonnutritive sucking

A nurse notes that an Eskimo woman does not cuddle or interact with her newborn other than to feed him, change his diapers or soiled clothes, and put him to bed. While evaluating this client's behavior with her infant, what realization does the nurse make? Select one: a. What appears to be a lack of interest in the newborn is, in fact, the cultural way of demonstrating intense love by attempting to ward off evil spirits. b. The woman needs a referral to a social worker for further evaluation of her parenting behaviors once she goes home with the newborn. c. The woman is inexperienced in caring for a newborn. d. Extra time needs to be planned for assisting the woman in bonding with her newborn.

A.....

Combination oral contraceptives, the vaginal ring and the patch contain two hormones. Which of the following is the primary hormone that stops ovulation? Select one: a. estrogen b. progesterone c. testosterone d. hCG

B

Despite warnings, prenatal exposure to alcohol continues to far exceed exposure to illicit drugs. Which condition is rarely associated with fetal alcohol syndrome (FAS)? Select one: a. Alcohol-related birth defects (ARBDs) b. Respiratory conditions c. Neural development disorder d. Intellectual impairment

B

Evidence-based practice (EBP) is the integration of the best: Select one: a. Randomized clinical trials, clinical expertise, and patients' requests b. Research evidence, clinical expertise, and patients' values c. Quantitative research, clinical expertise, and patients' preferences d. Research findings, clinical experience, and patients' preferences

B

The nurse should be cognizant of which condition related to skeletal injuries sustained by a neonate during labor or childbirth? Select one: a. Other than the skull, the most common skeletal injuries are to leg bones. b. Unless a blood vessel is involved, linear skull fractures heal without special treatment c. Clavicle fractures often need to be set with an inserted pin for stability. d. Newborn's skull is still forming and fractures fairly easily.

B

The obstetric provider has informed the nurse that she will be performing an amniotomy on the client to induce labor. What is the nurse's highest priority intervention after the amniotomy is performed? Select one: a. Taking the client's vital signs b. Assessing the fetal heart rate (FHR) c. Applying clean linens under the woman d. Performing a vaginal examination

B

What bacterial infection is definitely decreasing in neonates because of effective drug treatment? Select one: a. Escherichia coli infection b. Tuberculosis c. Group B streptococci (GBS) infection d. Candidiasis

C

The nurse is caring for a recently immigrated Chinese woman in the postpartum unit. Based on cultural beliefs and practices of the woman, the nurse would anticipate which of the following? (Select all that apply.) Select one or more: a. The woman prefers cold water for drinking. b. The woman prefers not to shower. c. The woman prefers to have her female relatives care for her baby. d. The woman prefers a wide variety of foods to eat.

C In traditional Chinese beliefs and practices, the woman is to rest and female family members take care of the infant. During the first month, the woman is to avoid yin energy by eating specific foods and avoiding drinking or touching cold water.

A nurse is teaching a woman about her menstrual cycle. The nurse states that ____ is the most important change that happens during the secretory phase of the menstrual cycle. Select one: a. Maturation of the graafian follicle b. Multiplication of the fimbriae c. Secretion of human chorionic gonadotropin d. Proliferation of the endometrium

D

Which nutritional recommendation regarding fluids is accurate? Select one: a. Water with fluoride is especially encouraged because it reduces the child's risk of tooth decay. b. Of the artificial sweeteners, only aspartame has not been associated with any maternity health concerns. c. Coffee should be limited to no more than 2 cups, but tea and cocoa can be consumed without worry. d. A woman's daily intake should be six to eight glasses of water, milk, and/or decaffeinated drinks.

D

While discussing the societal impacts of breastfeeding, the nurse should be cognizant of the benefits and educate the client accordingly. Which statement as part of this discussion would be incorrect? Select one: a. Breastfeeding requires fewer supplies and less cumbersome equipment. b. Breastfeeding benefits the environment. c. Breastfeeding saves families money. d. Breastfeeding costs employers in terms of time lost from work.

D. Breastfeeding costs employers in terms of time lost from work.

Abruptio placenta is a risk factor for amniotic fluid embolism. Select one: True False

True

A primiparous woman is watching her newborn sleep. She wants him to wake up and respond to her. The mother asks the nurse how much he will sleep every day. What is an appropriate response by the nurse? Select one: a. "The newborn sleeps approximately 17 hours a day, with periods of wakefulness gradually increasing." b. "He is being stubborn by not waking up when you want him to. You should try to keep him awake during the daytime so that he will sleep through the night." c. "He will only wake up to be fed, and you should not bother him between feedings." d. "He will probably follow your same sleep and wake patterns, and you can expect him to be awake soon."

a. "The newborn sleeps approximately 17 hours a day, with periods of wakefulness gradually increasing."

What is the highest priority nursing intervention when admitting a pregnant woman who has experienced a bleeding episode in late pregnancy? Select one: a. Assessing FHR and maternal vital signs b. Monitoring uterine contractions c. Performing a venipuncture for hemoglobin and hematocrit levels d. Placing clean disposable pads to collect any drainage

a. Assessing FHR and maternal vital signs CorrectCorrect!

A woman has requested an epidural block for her pain. She is 5 cm dilated and 100% effaced. The baby is in a vertex position and is engaged. The nurse increases the woman's IV fluid for a preprocedural bolus. Before the initiation of the epidural, the woman should be informed regarding the disadvantages of an epidural block. Which concerns should the nurse share with this client? (Select all that apply.) Select one or more: a. Higher body temperature may occur. b. Blood loss is not excessive. c. Ability to move freely is limited. d. Gastric emptying is not delayed. e. Orthostatic hypotension and dizziness may occur.

a. Higher body temperature may occur. c. Ability to move freely is limited. e. Orthostatic hypotension and dizziness may occur.

The nurse expects which maternal cardiovascular finding(s) during labor? (Select all that apply). Select one or more: a. Increased cardiac output b. Decreased pulse rate c. Decreased blood pressure d. Increased white blood cell (WBC) count

a. Increased cardiac output d. Increased white blood cell (WBC) count

A newborn is jaundiced and is receiving phototherapy via ultraviolet bank lights. What is the most appropriate nursing intervention when caring for an infant with hyperbilirubinemia and receiving phototherapy? Select one: a. Placing eye shields over the newborn's closed eyes b. Applying an oil-based lotion to the newborn's skin to prevent dying and cracking c. Changing the newborn's position every 4 hours d. Limiting the newborn's intake of milk to prevent nausea, vomiting, and diarrhea

a. Placing eye shields over the newborn's closed eyes

After assessing the FHR tracing shown below, which of the following interventions should the nurse perform? mc007-1.jpg - (Image of Late Decels) Select one: a. Turn the woman onto her side. b. Administer oxygen by nasal cannula. c. Encourage the patient to push with each contraction. d. Provide the patient with caring labor support.

a. Turn the woman onto her side.

Which of the following screens for cervical cancer? (Select all that apply). Select one or more: a. herpes simplex 1 and 2 b. high risk HPV test c. Pap smear test d. chlamydia

b. high risk HPV test c. Pap smear test

When is a prophylactic cerclage for an incompetent cervix usually placed (in weeks of gestation)? Select one: a. 23 to 24 b. After 24 c. 12 to 14 d. 6 to 8

c. 12 to 14

A 35-year-old G1 P1 postpartum woman is Rh negative and has given birth to an Rh positive infant. Rh0(D) immune globulin is to be administered. The most appropriate dose that the perinatal nurse would expect to be ordered would be: Select one: a. 120 mcg b. 250 mcg c. 300 mcg d. 350 mcg

c. 300 mcg

The clinic nurse is aware that the pregnant woman's blood volume increases by: Select one: a. 20% to 25% b. 30% to 35% c. 40% to 45% d. 50% to 55%

c. 40% to 45% Correct

Necrotizing enterocolitis (NEC) is an inflammatory disease of the gastrointestinal mucosa. The signs of NEC are nonspecific. What are generalized signs and symptoms of this condition? Select one: a. Hypertonia, tachycardia, and metabolic alkalosis b. Hypertension, absence of apnea, and ruddy skin color c. Abdominal distention, temperature instability, and grossly bloody stools d. Scaphoid abdomen, no residual with feedings, and increased urinary output

c. Abdominal distention, temperature instability, and grossly bloody stools

The obstetric nurse is preparing the client for an emergency cesarean birth, with no time to administer spinal anesthesia. The nurse is aware of and prepared for the greatest risks of administering general anesthesia to the client. What are these risks? (Select all that apply) Select one or more: a. Respiratory depression of the mother b. Uterine relaxation c. Aspiration of stomach contents d. Inadequate muscle relaxation e. Respiratory depression of the neonate f. Difficulty or inability to intubate

c. Aspiration of stomach contents Correct f. Difficulty or inability to intubate Correct

Which intervention can nurses use to prevent evaporative heat loss in the newborn? Select one: a. Placing the baby away from the outside walls and windows b. Keeping the baby out of drafts and away from air conditioners c. Drying the baby after birth, and wrapping the baby in a dry blanket d. Warming the stethoscope and the nurse's hands before touching the baby

c. Drying the baby after birth, and wrapping the baby in a dry blanket

A premature infant never seems to sleep longer than an hour at a time. Each time a light is turned on, an incubator closes, or people talk near her crib, she wakes up and inconsolably cries until held. What is the correct nursing diagnosis beginning with "ineffective coping, related to"? a. Severe immaturity b. Physiologic distress c. Environmental stress d. Behavioral responses

c. Environmental stress

A macrosomic infant is born after a difficult forceps-assisted delivery. After stabilization, the infant is weighed, and the birth weight is 4550 g (9 lb, 6 oz). What is the nurse's first priority? Select one: a. Perform a gestational age assessment to determine whether the infant is large for gestational age. b. Leave the infant in the room with the mother. c. Frequently monitor blood glucose levels, and closely observe the infant for signs of hypoglycemia. d. Immediately take the infant to the nursery.

c. Frequently monitor blood glucose levels, and closely observe the infant for signs of hypoglycemia.

A woman with severe preeclampsia has been receiving magnesium sulfate by intravenous infusion for 8 hours. The nurse assesses the client and documents the following findings: temperature of 37.1° C, pulse rate of 96 beats per minute, respiratory rate of 24 breaths per minute, BP of 155/112 mm Hg, 3+ DTRs, and no ankle clonus. The nurse calls the provider with an update. The nurse should anticipate an order for which medication? Select one: a. Magnesium sulfate bolus b. Calcium gluconate c. Hydralazine d. Diazepam

c. Hydralazine

According to professional standards (the Association of Women's Health, Obstetric and Neonatal Nurses [AWHONN], 2007), which action(s) cannot be performed by the nonanesthetist registered nurse who is caring for a woman with epidural anesthesia? (Select all that apply). Select one or more: a. Monitoring the status of the woman and fetus b. Replacing empty infusion bags with the same medication and concentrate c. Initiating epidural anesthesia d. Inject medication through the epidural catheter e. Stopping the infusion, and initiating emergency measures

c. Initiating epidural anesthesia Correct d. Inject medication through the epidural catheter Correct

The nurse is providing prenatal teaching to a group of diverse pregnant women. One woman, who indicates she smokes two to three cigarettes a day, asks about its impact on her pregnancy. The nurse explains that the most significant risk to the fetus is: Select one: a. Respiratory distress at birth b. Severe neonatal anemia c. Low neonatal birth weight d. Neonatal hyperbilirubinemia

c. Low neonatal birth weight CorrectCorrect!

According to research, which risk factor for PPD is likely to have the greatest effect on the client postpartum? Select one: a. Single-mother status b. Unplanned or unwanted pregnancy c. Prenatal depression d. Low socioeconomic status

c. Prenatal depression

When assessing the fetus using Leopold's maneuvers, the nurse feels a round, firm/hard, and movable fetal part in the fundal portion of the uterus and a long, smooth surface in the mother's right side close to midline. What is the position of the fetus? Select one: a. LOA b. LSP c. RSA d. ROA

c. RSA

What should the nurse's next action be if the client's white blood cell (WBC) count is 25,000/mm3 on her second postpartum day? Select one: a. Have the laboratory draw blood for reanalysis. b. Immediately begin antibiotic therapy. c. Recognize that this count is an acceptable range at this point postpartum d. Immediately inform the physician.

c. Recognize that this count is an acceptable range at this point postpartum CorrectCorrect!

During a routine prenatal visit in the third trimester, a woman reports she is dizzy and lightheaded when she is lying on her back. The most appropriate nursing action would be to: Select one: a. Order an EKG. b. Report this abnormal finding immediately to her care provider. c. Teach the woman to avoid lying on her back and to rise slowly because of supine hypotension. d. Order a nonstress test to assess fetal well-being.

c. Teach the woman to avoid lying on her back and to rise slowly because of supine hypotension.

Four babies have just been admitted into the neonatal nursery. Which of the babies should the nurse assess first? Select one: a. The baby with respirations 52, oxygen saturation 98% b. The baby with Apgar 9/9, weight 2960 grams c. The baby with temperature 96.3°F, length 17 inches d. The baby with glucose 60 mg/dL, heart rate 132

c. The baby with temperature 96.3°F, length 17 inches. Axillary temperature below 97.7°F is a sign of cold stress and must be treated immediately

A nurse is providing education to a support group of women newly diagnosed with breast cancer. It is important for the nurse to discuss which factor related to breast cancer with the group? Select one: a. Breast cancer is the leading cause of cancer death in women. b. In the United States, 1 in 10 women will develop breast cancer in her lifetime. c. The exact cause of breast cancer remains unknown. d. Genetic mutations account for 50% of women who will develop breast cancer.

c. The exact cause of breast cancer remains unknown.

What is the most common cause of birth defects in humans? Select one: a. Drugs b. Single gene mutations c. Unknown causes d. Viral infections

c. Unknown causes

At 1 minute after birth a nurse assesses an infant and notes a heart rate of 80 beats per minute, some flexion of extremities, a weak cry, grimacing, and a pink body but blue extremities. Which Apgar score does the nurse calculate based upon these observations and signs? Select one: a. 7 b. 6 c. 4 d. 5

d. 5

Which nursing action is most appropriate to correct a boggy uterus that is displaced above and to the right of the umbilicus? Select one: a. Notify the physician of an impending hemorrhage. b. Administer pitocin c. Assess the blood pressure and pulse. d. Assist the client in emptying her bladder.

d. Assist the client in emptying her bladder

Which neonatal complications are associated with hypertension in the mother? Select one: a. Hepatic or renal dysfunction b. Placental abruption and DIC c. Seizures and cerebral hemorrhage d. Intrauterine growth restriction (IUGR) and prematurity

d. Intrauterine growth restriction (IUGR) and prematurity

The nurse has evaluated a client with preeclampsia by assessing DTRs. The result is a grade of 3+. Which DTR response most accurately describes this score? Select one: a. Sluggish or diminished b. Active or expected response c. Brisk, hyperactive, with intermittent or transient clonus d. More brisk than expected, slightly hyperactive

d. More brisk than expected, slightly hyperactive

Women who are obese are at risk for several complications during pregnancy and birth. Which of these would the nurse anticipate with an obese client? (Select all that apply.) Select one or more: a. Cesarean birth b. Breech presentation c. Hypertension d. Wound infection e. Thromboembolism

A, C, D, E

Which adverse prenatal outcomes are associated with the HELLP syndrome? (Select all that apply.) Select one or more: a. Maternal and fetal death b. Cirrhosis c. Placental abruption d. Renal failure e. Placenta previa

A, C, E

The clinic nurse advocates for smoking cessation. Potential harmful effects of tobacco use include (select all that apply): Select one or more: a. Infertility impairment b. Fetal Alcohol Syndrome c. Insomnia d. Low birth weight of the fetus

A, D

A pregnant woman informs the nurse that her last normal menstrual period was on July 6, 2016. Using Naegele's rule, which of the following would the nurse determine to be the patient's estimated date of delivery (EDC)? Select one: a. January 9, 2017 b. April 13, 2017 c. April 20, 2017 d. September 6, 2017

B

The clinic nurse understands the meaning of the following terms related to pregnancy care. Match these terms with the definitions listed below: 1. Blotchy, brownish hyperpigmentation of the skin over the cheeks, nose and forehead. 2. Mother's experience that her baby has 'dropped' into the pelvis 3. Passive movement of the unengaged fetus 4. Pigmented line extending from the symphysis pubis to the top of the fundus 5. Stretch marks Choices: Melasma Ballottement Lightening Striae gravidarum Linea nigra

1. Melasma 2. Lightening 3. Ballottement 4. Linea nigra 5. Striae gravidarum

Match the definition with the correct term. 1: a pictorial analysis of an individual's chromosomes 2: this/these function(s) to inhibit cell growth and division cycle 3:the number of autosomes in humans 4: physical feature or psychological trait Options: 46 Karyotype Genotype Phenotype 22 Tumor Suppressor Genes Oncogenes

1: Karyotype 2: Tumor Suppressor Genes 3: 22 4: Phenotype Your answer is correct. The correct answer is: a pictorial analysis of an individual's chromosomes - karyotype, this/these function(s) to inhibit cell growth and division cycle - tumor supressor genes, the number of autosomes in humans - 22, physical feature or psychological trait - phenotype

Match the definition to the correct term. (HINT: not all terms are used). 1: number of maternal deaths from births and complications of pregnancy, childbirth and puerperium per 100.000 live births 2: number of deaths of infants younger than 1 year of age per 1000 births 3: number of live births in 1 year per 1000 population Options: Perinatal Mortality rate Maternal Mortality Rate Infant mortality Rate Birth Rate Neonatal Mortality Rate

1: Maternal Mortality Rate 2: Infant Mortality Rate 3: Birth Rate Your answer is correct. See Box 1-6, page 6 The correct answer is: number of maternal deaths from births and complications of pregnancy, childbirth and puerperium per 100.000 live births - maternal mortality rate, number of deaths of infants younger than 1 year of age per 1000 births - infant mortality rate, number of live births in 1 year per 1000 population - birth rate

Match the condition or trait with the appropriate mode of inheritance (more than one mode may be used and not all are used). 1: ovarian cancer 2: hemophilia 3: cleft lip 4: osteogenesis impertecta 5: sickle cell anemia Options: autosomal dominant X linked recessive multifactorial autosomal recessive X linked dominant

1: multifactorial 2: x-linked recessive 3: multifactorial 4: autosomal dominant 5: autosomal recessive You have correctly selected 4. See powerpoint presentation and Chapter 3 for feedback. Cancer is not hereditary- gene mutations can be. Cancer is multifactorial and starts with gene mutation. Other factors play a role in facilitating uncontrolled cell growth. The correct answer is: ovarian cancer - multifactorial inheritance, hemophilia - X linked recessive inheritance, cleft lip - multifactorial inheritance, osteogenesis impertecta - autosomal dominant inheritance, sickle cell anemia - autosomal recessive inheritance

A woman with gestational diabetes has had little or no experience reading and interpreting glucose levels. The client shows the nurse her readings for the past few days. Which reading signals the nurse that the client may require an adjustment of insulin or carbohydrates? Select one: a. 50 mg/dl just after waking up from a nap. This is too low; maybe eat a snack before going to sleep. b. 75 mg/dl before lunch. This is low; better eat now. c. 115 mg/dl 1 hour after lunch. This is a little high; maybe eat a little less next time. d. 115 mg/dl 2 hours after lunch. This is too high; it is time for insulin.

A

Alisson tells the nurse she is concerned her health insurance company will raise her premiums if they find out her baby tests positive for a single gene mutation, such as cystic fibrosis. The nurse tells Alisson about which of the following laws that protects her from such genetic discrimination? Select one: a. Genetic Information Non-discrimination Act (GINA) b. Genetic Protection Act (GPA) c. HIPAA d. There is no such law

A

An 18-year-old pregnant woman, gravida 1, para 0, is admitted to the labor and birth unit with moderate contractions every 5 minutes that last 40 seconds. The client states, "My contractions are so strong, I don't know what to do." Before making a plan of care, what should the nurse's first action be? Select one: a. Recognize that pain is personalized for each individual. b. Encourage the woman to lie on her side. c. Disturb the woman as little as possible. d. Assess for fetal well-being.

A

At a 2-month well-baby examination, it was discovered that an exclusively breastfed infant had only gained 10 ounces in the past 4 weeks. The mother and the nurse develop a feeding plan for the infant to increase his weight gain. Which change in dietary management will assist the client in meeting this goal? Select one: a. Have one extra breastfeeding session every 24 hours. b. Begin solid foods. c. Start iron supplements. d. Have a bottle of formula after every feeding.

A

Because a full bladder prevents the uterus from contracting normally, nurses intervene to help the woman spontaneously empty her bladder as soon as possible. If all else fails, what tactic might the nurse use? Select one: a. Inserting a sterile catheter b. Pouring water from a squeeze bottle over the woman's perineum c. Asking the physician to prescribe analgesic agents d. Placing oil of peppermint in a bedpan under the woman

A

Depo provera, or The Shot, contains which of the following hormones? Select one: a. progesterone b. estrogen and progesterone c. estrogen d. neither estrogen or progesterone

A

During a telephone follow-up conversation with a woman who is 4 days postpartum, the woman tells the nurse, "I don't know what's wrong. I love my son, but I feel so let down. I seem to cry for no reason!" Which condition might this new mother be experiencing? Select one: a. Postpartum blues b. Postpartum depression (PPD) c. Letting-go d. Attachment difficulty

A

During preconception counseling, the clinic nurse explains that the time period when the fetus is most vulnerable to the effects of teratogens occurs from: Select one: a. 2 to 8 weeks b. 4 to 12 weeks c. 5 to 10 weeks d. 6 to 15 weeks

A

Human immunodeficiency virus (HIV) may be transmitted perinatally or during the postpartum period. Which statement regarding the method of transmission is most accurate? Select one: a. Through the ingestion of breast milk from an infected mother b. From the use of unsterile instruments c. Only in the third trimester from the maternal circulation d. Only through the ingestion of amniotic fluid

A

If nonsurgical treatment for late PPH is ineffective, which surgical procedure would be appropriate to correct the cause of this condition? Select one: a. Dilation and curettage (D&C) b. Hysterectomy c. Laparotomy d. Laparoscopy

A

If the newborn has excess secretions, the mouth and nasal passages can be easily cleared with a bulb syringe. How should the nurse instruct the parents on the use of this instrument? Select one: a. Insert the compressed bulb into the center of the mouth. b. Suction the mouth first. c. Avoid suctioning the nares. d. Remove the bulb syringe from the crib when finished.

A

Parents are often asked if they would like to have an autopsy performed on their infant. Nurses who are assisting parents with this decision should be aware of which information? Select one: a. Some religions prohibit autopsy. b. In the current litigious society, more autopsies are performed than in the past. c. Autopsies are usually covered by insurance. d. Autopsies must be performed within a few hours after the infant's death.

A

The nurse is evaluating a neonate who was delivered 3 hours ago by vacuum-assisted delivery. The infant has developed a cephalhematoma. Which statement is most applicable to the care of this neonate? Select one: a. In many infants, signs of hemorrhage in a full-term infant are absent and diagnosed only through laboratory tests. b. Intracranial hemorrhage (ICH) as a result of birth trauma is more likely to occur in the preterm, low-birth-weight infant. c. Subarachnoid hemorrhage (the most common form of ICH) occurs in term infants as a result of hypoxia. d. Spinal cord injuries almost always result from vacuum-assisted deliveries.

A

The nurse is teaching a client with preterm premature rupture of membranes (PPROM) regarding self-care activities. Which activities should the nurse include in her teaching? Select one: a. Do not engage in sexual activity. b. Tampons are safe to use to absorb the leaking amniotic fluid. c. Report a temperature higher than 40° C. d. Taking frequent tub baths is safe.

A

The nurse knows that a FHR monitor printout indicates a Category III abnormal fetal heart rate pattern when: Select one: a. Baseline variability is minimal or absent with decelerations. b. FHR mirrors the uterine contractions. c. Occasional periodic accelerations occur. d. Baseline variability is 6 to 25 bpm with decelerations

A

The nurse should be cognizant of which physiologic effect of pain? Select one: a. Predominant pain of the first stage of labor is visceral pain that is located in the lower portion of the abdomen. b. Somatic pain of the second stage of labor is more generalized and related to fatigue. c. Pain during the third stage is a somewhat milder version of the pain experienced during the second stage. d. Referred pain is the extreme discomfort experienced between contractions.

A

The perinatal nurse notes a rapid decrease in the fetal heart rate that does not recover immediately following an amniotomy. The most likely cause of this obstetrical emergency is: Select one: a. Prolapsed umbilical cord b. Vasa previa c. Oligohydramnios d. Placental abruption

A

The perinatal nurse providing care to a laboring woman recognizes a Category II fetal heart rate tracing. The most appropriate initial action for uterine resuscitation is to: Select one: a. Assist the laboring woman to change her position b. Decrease the intravenous solution c. Request that the physician/certified nurse-midwife come to the hospital STAT d. Document the fetal heart rate and variability

A

The postpartum nurse caring for a 20-year-old G1 P1 woman who 3 hours ago delivered a healthy full-term infant, observes the woman who is lightly touching her baby girl with her fingertips but who seems to be uncomfortable holding her baby close to her body. Which of the following is an accurate interpretation of these observed behaviors? Select one: a. The woman is demonstrating behavior that facilitates infant attachment. b. The woman is in the taking-in phase. c. The woman is having difficulty in bonding with her baby. d. The woman needs to be medicated for pain.

A

Trina is 16 y/o and presents to the clinic with complaint of symptoms consistent with endometriosis. She does not want to get pregnant for several years. Which of the following treatments will the physician probably implement? Select one: a. OCPs b. GnRH agonist c. Surgery d. Danazol

A

What is the most important nursing action in preventing neonatal infection? Select one: a. Good handwashing or hand hygiene b. Isolation of infected infants c. Standard Precautions d. Separate gown technique

A

When assisting the mother, father, and other family members to actualize the loss of an infant, which action is most helpful? Select one: a. Ensuring the baby is clothed or wrapped if the parents choose to visit with the baby b. Making sure the family understands that naming the baby is important c. Setting a firm time for ending the visit with the baby so that the parents know when to let go d. Using the words lost or gone rather than dead or died

A

When caring for a pregnant woman with cardiac problems, the nurse must be alert for the signs and symptoms of cardiac decompensation. Which critical findings would the nurse find on assessment of the client experiencing this condition? Select one: a. Dyspnea, crackles, and an irregular, weak pulse b. Shortness of breath, bradycardia, and hypertension c. Increased urinary output, tachycardia, and dry cough d. Regular heart rate and hypertension

A

Which clinical finding or intervention might be considered the rationale for fetal tachycardia to occur? Select one: a. Maternal fever b. Regional anesthesia c. Magnesium sulfate administration d. Umbilical cord prolapse

A

Which statement is not an expected outcome for the client who attends a reputable childbirth preparation program? Select one: a. Childbirth preparation programs guarantee a pain-free childbirth. b. Childbirth preparation programs teach distraction techniques. c. Childbirth preparation programs increase the woman's sense of control. d. Childbirth preparation programs prepare a support person to help during labor

A

Which substance used during pregnancy causes vasoconstriction and decreased placental perfusion, resulting in maternal and neonatal complications? Select one: a. Tobacco b. Alcohol c. Caffeine d. Chocolate

A

Which type of formula is not diluted with water, before being administered to an infant? Select one: a. Ready-to-use b. Modified cow's milk c. Concentrated d. Powdered

A

Zita is 39 y/o G21002 who is 7 weeks pregnant. For which of the following is she at increased risk? Select one: a. offspring with chromosomal defect b. cephalo-pelvic disportion (CPD) c. postpartum hemorrhage d. anemia

A

The nurse is about to elicit the rooting reflex on a newborn baby. Which of the following responses should the nurse expect to see? Select one: a. When the cheek of the baby is touched, the newborn turns toward the side that is touched b. When the lateral aspect of the sole of the baby's foot is stroked, the toes extend and fan outward. c. When the baby is suddenly lowered or startled, the neonate's arms straighten outward and the knees flex. d. When the newborn is supine and the head is turned to one side, the arm on that same side extends.

A An infant exhibits Rooting reflex when the neonate turns his head toward the direction of the stimulus and opens his mouth. Choice 2 is the Babinski reflex; Choice 3 is the Startle or Moro reflex; Choice 4 is the Tonic Neck reflex

The nursery nurse notes the presence of diffuse edema on a baby girl's head. Review of the birth record indicates that her mother experienced a prolonged labor and difficult childbirth. By the second day of life, the edema has disappeared. The nurse documents the following condition in the infant's chart. Select one: a. Caput succedaneum b. Cephalhematoma c. Subperiosteal hemorrhage d. Epstein pearls

A Caput succedaneum is localized soft tissue edema of the scalp; feels spongy; may cross suture lines; results from prolonged pressure of the head against the maternal cervix during labor; resolves within the first week of life.

The nurse is teaching the parents of a female baby how to change a baby's diapers. Which of the following should be included in the teaching? Select one: a. Always wipe the perineum from front to back. b. Use an antibiotic ointment at the first sign of diaper rash c. Put powder on the buttocks every time the baby stools. d. Weigh every diaper in order to assess for hydration.

A Clean female genitals by washing from front to back to decrease the risk of cystitis

An ultrasound of a fetus' heart shows that "normal fetal circulation is occurring." Which of the following statements is consistent with the finding? Select one: a. Blood is seen moving from the right to left atria. b. Blood is returning to the placenta via the umbilical vein. c. Blood is returning to the right atrium from the pulmonary system. d. A right to left shunt is seen between the umbilical arteries.

A High levels of oxygenated blood enter the fetal circulatory system from the placenta vial the umbilical vein. The foramen ovale, an opening between the right and left atria, allow for oxygenated blood to be shunted right to left, or move from the right to left atria. Deoxygenated blood returns to the placenta via the umbilical arteries.

The nurse is working with a 36-year-old, married client, with 6 children who smokes. The woman states, "I don't expect to have any more kids, but I hate the thought of being sterile." Which of the following contraceptive methods would be best for the nurse to recommend to this client? Select one: a. Intrauterine contraceptive device (IUD) b. Contraceptive patch c. Bilateral tubal ligation d. Birth control pills with estrogen and progestin

A Intrauterine contraception device (IUD) is the recommended method for this patient. IUD has a low failure rate and provides long-term contraception for 3-10 years. Bilateral tubal ligation (BTL) is a surgical procedure which results in sterilization. Due to her history of smoking, neither birth control pills with estrogen nor contraceptive patch is recommended due to the increased risks for blood clots, heart disease, and strokes, also associated with smoking.

A woman with worsening preeclampsia is admitted to the hospital's labor and birth unit. The physician explains the plan of care for severe preeclampsia, including the induction of labor, to the woman and her husband. Which statement by the husband leads the nurse to believe that the couple needs further information? Select one: a. "Since we will be here for a while, I will call my mother so she can bring the two boys—2 years and 4 years of age—to visit their mother." b. "I will give my wife ice chips to eat during labor." c. "I will help my wife use the breathing techniques that we learned in our childbirth classes." d. "I will stay with my wife during her labor, just as we planned."

A Need to maintain quiet, low stimulant environment.

Which statement correctly describes the nurse's responsibility related to electronic fetal monitoring (EFM)? Select one: a. Teach the woman and her family about the monitoring equipment and discuss any questions they have. b. Report abnormal findings to the care provider before initiating corrective actions. c. Inform the support person that the nurse will be responsible for all comfort measures when the electronic equipment is in place. d. Document the frequency, duration, and intensity of contractions measured by the external device.

A Nurses are expected to independently assess, interpret, and intervene related to interpretations of electronic fetal monitoring (EFM). Nurses are expected to provide information and explanations to patients and their families in order to relieve anxiety and answer questions. Nurses are expected to share information with obstetric providers to insure clear communication. EFM is a means to information regarding fetal response to labor but does not take the place of nursing and support person interactions. Remember that external devices cannot measure uterine intensity/pressure during a contraction but can provide information on frequency and duration.

A nurse working with an infertile couple has made the following nursing diagnosis: Sexual dysfunction related to decreased libido. Which of the following assessments is the likely reason for this diagnosis? Select one: a. The couple has established a set schedule for their sexual encounters. b. The couple has been married for more than 8 years. c. The couple lives with one set of parents. d. The couple has close friends who gave birth within the last year.

A Pregnancy occurs with ovulation; sexual intercourse outside the time of ovulation will not result in conception.

During the postpartum assessment, the perinatal nurse notes that a patient who has just experienced a forceps-assisted birth now has a large quantity of bright red bleeding. Her uterine fundus is firm and midline. The nurse's most appropriate action is to notify the physician/certified nurse midwife and anticipate the need for: Select one: a. Vaginal assessment and possible repair of vaginal and/or cervical lacerations b. An oxytocin infusion c. Further information for the woman/family about forceps d. Bladder assessment and catheterization

A Risks as a result of forceps delivery include vaginal/cervical lacerations, extension of episiotomy, hemorrhage related to uterine atony or rupture, perineal hematoma, bladder injury, and perineal wound infection.

Which of the following providers would be best for Lisa (7 weeks pregnant) to receive prenatal care services? She is a G40211 with a history of preterm births, one stillbirth, one cesarean section, and high blood pressure during pregnancy. Select one: a. Physician b. Direct Entry Midwife c. Doula d. Lay Midwife

A She has a hx of High Risk Pregnancies

An ethical dilemma unique to perinatal nursing is the: Select one: a. Embryo/fetal rights b. Intensive use of technology to maintain life c. Limited access to care d. Ethnicity disparities in health care

A The correct answer is: Embryo/fetal rights

Which of the following is correct regarding endometriosis? Select one: a. The physical symptoms of endometriosis can affect the woman's mental health. b. The abnormal tissue bleeds into surrounding tissue during the secretory stage of the menstrual cycle. c. Endometriosis causes severe headaches. d. Metronidazole is used to treat endometriosis.

A The physical symptoms of endometriosis can have an effect on the woman's mental health; she may experience anger and grief related to loss of fertility and the pain related to this condition can interfered with her social activities. Dyspareunia can have an effect of intimate relationships. Abnormal tissue breaks down and bleeds into the surrounding tissues during the menstrual phase. Endometriosis is not associated with headaches and the drugs used in treatment include Danazol, Lupron, oral contraceptives, and progestins.

When the nurse is alone with a battered client, the client seems extremely anxious and says, "It was all my fault. The house was so messy when he got home, and I know he hates that." What is the most suitable response by the nurse? Select one: a. "No one deserves to be hurt. It's not your fault. How can I help you?" b. "He will never find out what we talk about. Don't worry. We're here to help you." c. "What else do you do that makes him angry enough to hurt you?" d. "You have to remember that he is frustrated and angry so he takes it out on you."

A Violence against women is not acceptable in any situation; no deserves to be hurt or mistreated. Telling her this may be the first step to empower her. see p107 of book. Your answer is correct. The correct answer is: "No one deserves to be hurt. It's not your fault. How can I help you?"

Painful nipples are a major reason why women stop breastfeeding. A primary intervention to decrease nipple irritation is: Select one: a. Teaching proper techniques for latching-on and releasing of suction b. Applying hot compresses to breast prior to feeding c. Instructing woman to express colostrum or milk at the end of the feeding session and rub it on her nipples d. Air drying nipples for 10 minutes at the end of the feeding session

A While all these interventions are correct, the primary intervention is to ensure correct latching-on and suction release as problems with these lead to early cessation of breastfeeding.

Which term is an accurate description of the process by which people retain some of their own culture while adopting the practices of the dominant society? Select one: a. Acculturation b. Cultural relativism c. Ethnocentrism d. Assimilation

A Your answer is correct. See pp 22-23 The correct answer is: Acculturation

What is (are) the most critical nursing action(s) in caring for the newborn immediately after the birth? (Select all that apply). Select one or more: a. Keeping the airway clear b. Drying and warming the newborn. c. Fostering parent-newborn attachment d. Administering eye drops and vitamin K

A & B a. Keeping the airway clear If only one answer were allowed this would be it; since more than one answer is allowed, drying and warming the neonate are also top priority. b. Drying and warming the newborn. Warming by putting baby on mother's abdomen and covered with warm blanket, or baby is wrapped in warm blanket.

Which finding on a prenatal visit at 10 weeks of gestation might suggest a hydatidiform mole? (Select all that apply). Select one or more: a. Complaint of severe nausea b. Fundal height measurement of 18 cm c. History of bright red spotting for 1 day, weeks ago d. Blood pressure of 120/80 mm Hg

A, B

Which of the following are milestones in the care of mothers and babies? (Select all that apply). Select one or more: a. abortion is legalized b. folic acid is recommended to prevent neural tube defects c. HIV vaccine is available d. FDA approves birth control pill for men

A, B

Which of the following hormones basically tell the brain to release the egg during the menstrual cycle? (Select all that apply). Select one or more: a. Luteinizing hormone b. Follicle-stimulating hormone c. Prostaglandins d. Progesterone

A, B

Which of the following is true about sperm? (select all that apply). Select one or more: a. Sperm can live up to 72 hours in the female reproductive tract b. There are approx 200-500 million sperm with each ejaculation c. All of the sperm a male will have are developed during prenatal development d. Sperm undergo primary mitosis in the testes e. Sperm are formed in the vas deferens

A, B

The clinic nurse teaches expectant mothers about the differences between breast milk and commercially prepared infant formulas. When compared to commercially prepared formulas, breast milk has (select all that apply): Select one or more: a. More carbohydrates b. Less protein c. Fewer nutrients d. Less cholesterol

A, B Human breast milk contains more carbohydrates, less protein, and more cholesterol than cow's milk or infant formulas. Commercially prepared infant formulas use vegetable oils which are void of cholesterol.

The perinatal nurse describes infant feeding cues to a new mother. These feeding cues include (select all that apply): Select one or more: a. Awake and alert b. Mouth movements c. Moving the hand to the mouth d. Yawning

A, B, C

The perinatal nurse is caring for Christy following the birth of her first child. Based on Christy's history, the RN recognizes that risk factors for postpartum depression include: Select one or more: a. Loss of friends based on upcoming divorce; family is unable to assist b. Separated from spouse pending divorce c. Unplanned cesarean delivery secondary to Category III fetal tracing d. Good prenatal care with uneventful pregnancy

A, B, C

Which societal factors have a strong influence on parental response to their infant? (Select all that apply.) Select one or more: a. Mothers older than 35 years of age often deal with more stress related to work and career issues, as well as decreasing libido. b. An adolescent mother's egocentricity and unmet developmental needs interfere with her ability to parent effectively. c. An adolescent mother is likely to use less verbal instruction, be less responsive, and interact less positively than other mothers. d. Adolescent mothers have a higher documented incidence of child abuse. e. Relationships between adolescent mothers and fathers are more stable than older adults.

A, B, C

Which of the following are true regarding chlamydial infection in young women? (Select all that apply). Select one or more: a. Women ages 15-24 have the highest rates of infection b. it can increase risk for infertility and ectopic pregnancy c. women can have it and have no symptoms d. it is not curable

A, B, C See pp 149-150 of text

The perinatal nurse screens all pregnant women early in pregnancy for maternal attachment risk factors, which include (select all that apply): Select one or more: a. Adolescence b. Low educational level c. History of depression d. A strong support system for the pregnancy

A, B, C Maternal attachment to the fetus is an important area to assess and can be useful in identifying families at risk for maladaptive behaviors. The nurse should assess for indicators such as unintended pregnancy, domestic violence, difficulties in the partner relationship, sexually transmitted infections, limited financial resources, substance use, adolescence, poor social support systems, low educational level, the presence of mental conditions, or adolescence that might interfere with the patient's ability to bond with and care for the infant. A strong support system can facilitate the patient's ability to bond with and care for the infant.

Heat loss through radiation can be reduced by: (Select all that apply). Select one or more: a. Removing wet blankets used to dry the infant after birth from the crib b. Placing a stocking cap on the neonate's head and placing in a crib c. Providing skin-to-skin contact with the mother and covering both with a warm blanket when the room temperature is cool d. Placing crib near a warm wall

A, B, C, D

Pre-surgical nursing care for the woman who is going to have a hysterectomy for ovarian cancer includes which of the following? (Select all that apply). Select one or more: a. identification band is in place b. administer enema if ordered c. teach post-op routine care such as need for early ambulation d. ensure lab results are available to the surgeon prior to surgery e. Sips of water for hydration

A, B, C, D

The nurse is teaching the parents of a healthy newborn about infant safety. Which of the following should be included in the teaching plan? (Select all that apply). Select one or more: a. Water temperature for the infant's bath should be 100.4 degrees F. b. Do not cook while holding an infant c. Cover electrical outlets d. Remove strings from infant sleepwear, bedding, and pacifiers to prevent strangulation.

A, B, C, D

Ultrasound can be used in antepartum care for which of the following assessments? (Select all that apply). Select one or more: a. fetal growth b. gestational age c. adjunct use in chorionic villi sampling (CVS) d. placental position and function

A, B, C, D

Which of the following are ethical issues regarding genetics/genomics? (Select all that apply). Select one or more: a. fairness and use of genetic information b. discrimination, psychological impact, and stigmatization regarding one's genome c. informed consent in reproductive issues and genetic testing d. privacy and confidentiality of genetic information

A, B, C, D

Which of the following are indications for cesarean birth? (Select all that apply.) Select one or more: a. Previous cesarean birth b. Placental abnormalities c. Previous uterine surgery d. Failure to progress in labor

A, B, C, D

Which of the following are management options for couples faced with infertility challenges, depending of the cause? (Select all that apply). Select one or more: a. remain child-free b. Medication c. adoption d. assisted reproductive technology procedures

A, B, C, D

Which of the following would the nurse explain to the patient about fertilization? (Select all that apply). Select one or more: a. The best place for fertilization to occur is in the ampulla of the fallopian tube b. After the sperm enters the egg, the egg becomes impenetrable to other sperm c. The sperm must travel up the correct fallopian tube to fertilize the egg d. After fertilization, the chromosomes combine for a total of 46

A, B, C, D

Secondary amenorrhea results from (select all that apply): Select one or more: a. Polycystic ovary syndrome . b. Uncontrolled Diabetes c. Emotional distress d. Pregnancy

A, B, C, D A: Secondary amenorrhea is no menses in 6 months in a woman who has had normal menstrual cycles. May result from: lack of ovarian production, pregnancy, polycystic ovary syndrome, nutritional and endocrine disturbances, uncontrolled diabetes, heavy athletic activity, or emotional distress D: Nutritional disturbances such as anoxia and emotional distress can cause secondary amenorrhea.

Nursing actions focused at reducing a postpartum woman's risk for cystitis include which of the following? (Select all that apply.) Select one or more: a. Voiding within a few hours post-birth b. Oral intake of a minimum of 3000 mL per day c. Changing peri-pads every 3 to 4 hours or more frequently as indicated d. Reminding the woman to void every 3 to 4 hours while awake

A, B, C, D Correct - Early voiding helps flush bacteria from the urethra. Voiding every 3 to 4 hours will decrease the risk of bacterial growth in the bladder. Soiled peri-pads are a media for bacterial growth. It is recommend that a postpartum woman drink a minimum of 3000 mL/day to help dilute urine and promote frequent voiding.

The client is undergoing treatment for ovarian cancer. Which common nutritional problems are related to gynecologic cancers and the treatment thereof? (Select all that apply.) Select one or more: a. Constipation b. Increased Appetite c. Diarrhea d. Nausea and vomiting e. Stomatitis

A, C, D, E

Jennifer is 3 hours postpartum following the vaginal delivery of a 9lb 15oz baby girl. Estimated blood loss at delivery was 800 ml. The RN is aware that Jennifer experienced an early-postpartum hemorrhage. Select the appropriate nursing actions for the care of this patient. (Select all that apply.) Select one or more: a. Maintain IV site in case fluids/medication for PPH are indicated b. Frequent fundal assessment to prevent uterine atony and further blood loss c. Assess for displaced uterus secondary to overdistended bladder. d. Assess lochia for amount and for clots

A, B, C, D Correct - PPH is blood loss greater than 500 ml for vaginal deliveries and 1000 ml for cesarean with a 10% drop in hemoglobin and/or hematocrit. Unfortunately, postpartum women may not show signs/symptoms of PPH until about 1/3 of entire blood volume is lost. RNs must frequently assess uterine tone, location, and position as well as blood loss amount and characteristics (slow, steady, sudden, massive, presence of clots, possible distended bladder). Keeping the IV site intact will allow immediate access to fluids and/or medications should PPH worsen.

During life in utero, oxygenation of the fetus occurs through transplacental gas exchange. When birth occurs, four factors combine to stimulate the respiratory center in the medulla. The initiation of respiration then follows. What are these four essential factors? Select one or more: a. Mechanical b. Sensory c. Chemical d. Psychologic e. Thermal

A, B, C, E

A nurse is discussing the signs and symptoms of mastitis with a mother who is breastfeeding. Which findings should the nurse include in the discussion? (Select all that apply.) Select one or more: a. Warmth in the breast b. Fever and flulike symptoms c. Small white blister on the tip of the nipple d. Breast tenderness

A, B, D

A couple who has been attempting to become pregnant for 5 years is seeking assistance from an infertility clinic. The nurse assesses the clients' emotional responses to their infertility. Which of the following responses would the nurse expect to find? (Select all that apply.) Select one or more: a. Anger at others who have babies. b. Feelings of failure because they cannot make a baby.. c. Sexual excitement because they want to conceive a baby. d. Guilt on the part of one partner because he or she is unable to give the other a baby.

A, B, D Infertility can be seen as a crisis in the couple's lives and relationship. The diagnosis and treatment of infertility can cause anger with others who have babies, feelings of failure, and feelings of guile. Sexual dysfunction can occur based on the type of infertility testing required and the method of treatment.

A pregnant woman was admitted for induction of labor at 43 weeks of gestation with sure dates. A nonstress test (NST) in the obstetrician's office revealed a nonreactive tracing. On artificial rupture of membranes, thick meconium-stained fluid was noted. What should the nurse caring for the infant after birth anticipate? (Select all that apply). Select one or more: a. dry, cracked skin b. meconium aspiration c. excessive vernix caseosa covering the skin d. absence of scalp hair, e. hypoglycemia f. increased amount of subcutaneous fat

A, B, E

Which congenital anomalies can occur as a result of the use of antiepileptic drugs (AEDs) in pregnancy? (Select all that apply.) Select one or more: a. Neural tube defects b. Congenital heart disease c. Gastroschisis d. Diaphragmatic hernia e. Cleft lip

A, B, E

27 year old Amy G0 presents to the clinic for a preconceptual visit. Her BMI is 41. What would the nurse include in their education for Amy about pregnancy and a high BMI? (Select all that apply). Select one or more: a. There is increased risk for blood clot in her legs and lungs. b. There is decreased risk for cesarean section. c. There is increased risk for gestational diabetes. d. There is increased risk for preterm birth. e. There is decreased risk for miscarriage.

A, C, D

A nurse is discussing the signs and symptoms of mastitis with a mother who is breastfeeding. Which findings should the nurse include in the discussion? (Select all that apply.) Select one or more: a. Fever and flulike symptoms b. Small white blister on the tip of the nipple c. Breast tenderness d. Warmth in the breast

A, C, D

The nurse is caring for a postpartum woman who gave birth to a healthy, full-term baby girl. She has a 2-year-old son. She voices concern about her older child's adjustment to the new baby. Nursing actions that will facilitate the older son's adjustment to having a new baby in the house would include which of the following? (Select all that apply.) Select one or more: a. Explain to the mother that she can have her son visit her in the hospital. b. Teach her son how to change the baby's diapers. c. Assist her son in holding his new baby sister. d. Recommend that she spend time reading to her older son while he sits in her lap.

A, C, D

The nurse is explaining to 20 y/o Dana about symptoms that need to be reported to her health care provider. These include which of the following? (select all that apply). Select one or more: a. vaginal bleeding b. occasional headaches relieved with rest and hydration c. Fever d. absence of fetal movements after quickening

A, C, D

Which of the following are common symptoms for gonorrhea? Select all that apply. Select one or more: a. pelvic pain b. painful ulcer(s) c. rectal discharge d. no symptoms

A, C, D

25 year old Susan (G0000) is thinking about getting pregnant this next year, but wants an effective method right now. She currently uses condoms. She denies medical problems. Which of the following birth control methods are most appropriate for the nurse to discuss with Susan? (Select all that apply). Select one or more: a. Oral contraceptive pills b. Copper IUD (Paragard) c. The birth control patch d. Condoms (continue) e. The implant

A, C, D A: Long-acting contraception is not the best choice for a woman who desires pregnancy within one year. More appropriate methods include birth control pills and the patch, and the vaginal ring as they are not long acting and are reversible once the woman stops using them. Condoms are short acting with no hormones. C: Long-acting contraception is not the best choice for a woman who desires pregnancy within one year. More appropriate methods include birth control pills and the patch, and the vaginal ring as they are not long acting and are reversible once the woman stops using them. Condoms are short acting with no hormones. D: Long-acting contraception is not the best choice for a woman who desires pregnancy within one year. More appropriate methods include birth control pills and the patch, and the vaginal ring as they are not long acting and are reversible once the woman stops using them. Condoms are short acting with no hormones.

Indications for a primary cesarean birth are often nonrecurring. Therefore, a woman who has had a cesarean birth with a low transverse scar may be a candidate for vaginal birth after cesarean (VBAC). Which clients would be less likely to have a successful VBAC? (Select all that apply.) Select one or more: a. African-American race b. Lengthy interpregnancy interval c. Estimated fetal weight <4000 g d. Maternal obesity (BMI >30) e. Delivery at a rural hospital

A, D, E

What are some common characteristics of a potential male batterer? (select all that apply) Select one or more: a. Personality disorders b. High frustration tolerance c. High level of self-esteem d. Ability to empathize with others e. Substance abuse problems

A, E

Which of the following infections are included in the acronym, TORCH? (Select all that apply). Select one or more: a. Rubella b. Chlamydia c. Influenza d. Zika Virus e. Syphilis

A, E

Which of the following is effective treatment for dysmenorrhea? (Select all that apply). Select one or more: a. Oral contraceptive pills b. SSRIs c. Antibiotics d. Morphine e. NSAIDs

A, E

A new father is ready to take his wife and newborn son home. He proudly tells the nurse who is discharging them that within the next week he plans to start feeding the infant cereal between breastfeeding sessions. Which information should the nurse provide regarding this feeding plan? Select one: a. "Feeding solid foods between breastfeeding sessions before your son is 4 to 6 months old will lead to an early cessation of breastfeeding." b. "Your feeding plan will help your son sleep through the night." c. "Feeding solid foods before your son is 4 to 6 months old will limit his growth." d. "Feeding solid foods before your son is 4 to 6 months old may decrease your son's intake of sufficient calories."

A. "Feeding solid foods between breastfeeding sessions before your son is 4 to 6 months old will lead to an early cessation of breastfeeding."

A new mother asks the nurse what the "experts say" about the best way to feed her infant. Which recommendation of the American Academy of Pediatrics (AAP) regarding infant nutrition should be shared with this client? Select one: a. Infants should be given only human milk for the first 6 months of life. b. Infants fed on formula should be started on solid food sooner than breastfed infants. c. After 6 months, mothers should shift from breast milk to cow's milk. d. If infants are weaned from breast milk before 12 months, then they should receive cow's milk, not formula

A. Infants should be given only human milk for the first 6 months of life

Because of the premature infant's decreased immune functioning, what nursing diagnosis should the nurse include in a plan of care for a premature infant? a. Risk for infection b. Delayed growth and development c. Ineffective thermoregulation d. Ineffective infant feeding pattern

A. Risk for Infection

A 25-year-old gravida 3, para 2 client gave birth to a 9-pound, 7-ounce boy, 2 hours ago after augmentation of labor with oxytocin (Pitocin). She presses her call light, and asks for her nurse right away, stating "I'm bleeding a lot." What is the most likely cause of postpartum hemorrhaging in this client? Select one: a. Retained placental fragments b. Uterine atony c. Puerperal infection d. Unrepaired vaginal lacerations

B

A new mother asks the nurse what the "experts say" about the best way to feed her infant. Which recommendation of the American Academy of Pediatrics (AAP) regarding infant nutrition should be shared with this client? Select one: a. After 6 months, mothers should shift from breast milk to cow's milk. b. Infants should be given only human milk for the first 6 months of life. c. If infants are weaned from breast milk before 12 months, then they should receive cow's milk, not formula. d. Infants fed on formula should be started on solid food sooner than breastfed infants.

B

A nurse is providing breast care education to a client after mammography. Which information regarding fibrocystic changes in the breast is important for the nurse to share? Select one: a. Lumpiness is accompanied by tenderness after menses. b. Healthy women with fibrocystic breast disease find lumpiness with pain and tenderness in varying degrees in the breast tissue during menstrual cycles. c. It is a premalignant disorder characterized by lumps found in the breast tissue. d. Fibrocystic breast disease is a disease of the milk ducts and glands in the breasts

B

17 y/o Suzanna presents to the clinic with complaint of late menses by 5 days. She denies any health problems and states her periods are every 4 weeks. She wants a pregnancy test. She states she does not trust the negative home pregnancy test. The nurse performs the pregnancy test and it is negative. The nurse knows the provider will recommend which of the following actions? Select one: a. Schedule a pelvic ultrasound to confirm negative pregnancy test b. Recommend Suzanna repeat the urine pregnancy test in one week if no period by that time. Assess for contraceptive use. c. Order a blood pregnancy test d. Prepare the patient for a pelvic examination, to include pap smear and testing for STDs

B

A pregnant woman is at 38 weeks of gestation. She wants to know whether there are any signs that "labor is getting close to starting." Which finding is an indication that labor may begin soon? Select one: a. Weight gain of 1.5 to 2 kg (3 to 4 lb) b. Surge of energy c. Urinary retention d. Increase in fundal height

B

A pregnant woman wants to breastfeed her infant; however, her husband is not convinced that there are any scientific reasons to do so. The nurse can give the couple printed information comparing breastfeeding and bottle feeding. Which statement regarding bottle feeding using commercially prepared infant formulas might influence their choice? Select one: a. Commercially prepared formula ensures that the infant is getting iron in a form that is easily absorbed. b. Bottle feeding using a commercially prepared formula increases the risk that the infant will develop allergies. c. Bottle feeding helps the infant sleep through the night. d. Bottle feeding requires that multivitamin supplements be given to the infant.

B

A woman at 24 weeks of gestation states that she has a glass of wine with dinner every evening. Why would the nurse counsel the client to eliminate all alcohol? Select one: a. The fetus is at risk for multiple organ anomalies. b. The fetus is placed at risk for altered brain growth. c. She will be at risk for abusing other substances as well. d. Daily consumption of alcohol indicates a risk for alcoholism.

B

A woman presents for prenatal care at 6 weeks' gestation by LMP. Which of the following findings would the nurse expect to see? Select one: a. Multiple pillow orthopnea b. Maternal ambivalence c. Fundus at the umbilicus d. Pedal and ankle edema

B

A woman who is 30 weeks of gestation arrives at the hospital with bleeding. Which differential diagnosis would not be applicable for this client? Select one: a. Placenta previa b. Spontaneous abortion c. Abruptio placentae d. Cord insertio

B

After the Nurse Practitioner treats a 25 y/o's Bartholin cyst with incision and drainage procedure (I&D), the nurse is providing take-home instructions. The patient asks, "Will this happen again?" The nurse's best response is: Select one: a. "No, since it is drained it won't come back." b. "Bartholin cysts can recur. There is another procedure that can be done to reduce risk for recurrence.Would you like to talk to the NP about it?" c. "I cannot answer that question for you. Let me have the NP come and talk to you." d. "If you take your antibiotics and not get an infection, recurrence is rare. Let's talk about how to prevent infection."

B

An 18-year-old client who has reached 16 weeks of gestation was recently diagnosed with pregestational diabetes. She attends her centering appointment accompanied by one of her girlfriends. This young woman appears more concerned about how her pregnancy will affect her social life than her recent diagnosis of diabetes. A number of nursing diagnoses are applicable to assist in planning adequate care. What is the most appropriate diagnosis at this time? Select one: a. Risk for injury, to the fetus related to birth trauma b. Deficient knowledge, related to diabetic pregnancy management c. Deficient knowledge, related to insulin administration d. Risk for injury, to the mother related to hypoglycemia or hyperglycemia

B

Angela is 12 weeks pregnant in October. She asks the nurse if she should get the flu shot. The nurse's best response would be: Select one: a. I cannot tell you. You need to ask your physician. b. It is highly recommended you get the flu shot as it is more dangerous for you and your baby to get the flu during pregnancy than to get the shot. c. You should avoid all immunizations during pregnancy. d. You should wait until you are at least 20 weeks pregnant.

B

Having a genetic mutation may create an 85% chance of developing breast cancer in a woman's lifetime. Which condition does not increase a client's risk for breast cancer? Select one: a. Cowden syndrome b. Paget disease c. BRCA1 or BRCA2 gene mutation d. Li-Fraumeni syndrome

B

In assisting the breastfeeding mother to position the baby, which information regarding positioning is important for the nurse to keep in mind? Select one: a. Women with perineal pain and swelling prefer the modified cradle position. b. Whatever the position used, the infant is "belly to belly" with the mother. c. The cradle position is usually preferred by mothers who had a cesarean birth. d. While supporting the head, the mother should push gently on the occiput.

B

In which clinical situation would the nurse most likely anticipate a fetal bradycardia? Select one: a. Intraamniotic infection b. Prolonged umbilical cord compression c. Fetal anemia d. Tocolytic treatment using terbutaline

B

Information provided by the nurse that addresses the function of the amniotic fluid is that the amniotic fluid helps the fetus to maintain a normal body temperature and also: Select one: a. Facilitates asymmetrical growth of the fetal limbs b. Cushions the fetus from mechanical injury c. Promotes development of muscle tone d. Produces stool in the fetal intestines

B

Intimate partner violence (IPV) against women consists of actual or threatened physical or sexual violence and psychological and emotional abuse. Screening for IPV during pregnancy is recommended for: Select one: a. Pregnant women with a history of domestic violence b. All pregnant women c. All low-income pregnant women d. Pregnant adolescents

B

Karen, a G2 P2, experienced a precipitous birth 90 minutes ago. Her infant is 4200 grams and a repair of a second-degree laceration was needed following the birth. As part of the nursing assessment, the nurse discovers that Karen's uterus is boggy. Furthermore, it is noted that Karen's vaginal bleeding has increased. The nurse's most appropriate first action is to: Select one: a. Assess vital signs including blood pressure and pulse. b. Massage the uterine fundus with continual lower segment support. c. Measure and document each perineal pad changed in order to assess blood loss. d. Ensure appropriate lighting for a perineal repair if it is needed.

B

Mary G10000 is 12 weeks pregnant in October. She is not sure if she wants the flu shot. What is the best message the nurse should include in their education for Mary? Select one: a. If pregnant women get the flu, they usually don't get seriously ill. The flu shot is a good idea for women to get. b. If pregnant women get the flu, they can get seriously ill, require hospitalization and possibly die. We recommend you get the flu shot because it is a safe protective action to avoid these bad outcomes. c. If pregnant women get the flu, they can get sick, require hospitalization and possibly die. It is really up to you to get the flu shot. d. If pregnant women get the flu, they get a little ill. It is OK if you choose not to get flu shot even if it is safe.

B

Maternal nutritional status is an especially significant factor of the many that influence the outcome of pregnancy. Why is this the case? Select one: a. Women love obsessing about their weight and diets b. Adequate nutrition is an important preventive measure for a variety of problems. c. Maternal nutritional status is extremely difficult to adjust because of an individual's ingrained eating habits. d. A woman's preconception weight becomes irrelevant.

B

Nurses should be able to teach breastfeeding mothers the signs that the infant has correctly latched on. Which client statement indicates a poor latch? Select one: a. "I feel a firm tugging sensation on my nipples but not pinching or pain." b. "I hear a clicking or smacking sound." c. "My baby's jaw glides smoothly with sucking." d. "My baby sucks with cheeks rounded, not dimpled."

B

On day 3 of life, a newborn continues to require 100% oxygen by nasal cannula. The parents ask if they may hold their infant during his next gavage feeding. Considering that this newborn is physiologically stable, what response should the nurse provide? Select one: a. "You may only hold your baby's hand during the feeding." b. "You may hold your baby during the feeding." c. "Feedings cause more physiologic stress; therefore, the baby must be closely monitored. I don't think you should hold the baby." d. "Parents are not allowed to hold their infants who are dependent on oxygen."

B

Postoperative care of the pregnant woman who requires abdominal surgery for appendicitis includes which additional assessment? Select one: a. Intake and output (I&O) and intravenous (IV) site b. Fetal heart rate (FHR) and uterine activity c. Vital signs and incision d. Signs and symptoms of infection

B

Preeclampsia begins at what point during the pregnancy? Select one: a. At 12 weeks gestation b. At placental implantation c. At conception d. After 20 weeks gestation

B

Primary prevention of Pelvic inflammatory disease or PID, includes which of the following? Select one or more: a. Get early treatment of PID and take all antibotics b. Avoid getting STDs, such as chlamydia and gonorrhea c. Never kiss despite both partners tests negative for STD d. Get routine testing for chlamydia

B

Screening at 24 weeks of gestation reveals that a pregnant woman has gestational diabetes mellitus (GDM). In planning her care, the nurse and the client mutually agree that an expected outcome is to prevent injury to the fetus as a result of GDM. This fetus is at the greatest risk for which condition? Select one: a. Congenital anomalies of the central nervous system b. Macrosomia c. Preterm birth d. Low birth weight

B

Susie is 9 weeks pregnant (based on her last menstrual period) on her first prenatal visit. She tells the nurse she smoked marijuana approx 2 months ago but denies any use since then. She asks "will this hurt my baby?" The nurse's best response is: Select one: a. "I do not know. You will need to talk to your physician about that." b. "Based on your last menstrual period, the risk of harm to your baby from smoking marijuana at that time is very low." c. "That would have been during the most vulnerable time for fetal development so yes, there might be risk to your baby." d. "Yes, your baby might have significant birth defects from smoking marijuana at that time."

B

The clinic nurse talks with Kathy about her possible pregnancy. Kathy has experienced amenorrhea for 2 months, nausea during the day with vomiting every other morning, and breast tenderness. These symptoms are best described as: Select one: a. Positive signs of pregnancy b. Presumptive signs of pregnancy c. Probable signs of pregnancy d. Possible signs of pregnancy

B

The perinatal nurse teaches the postpartum woman about warning signs regarding development of postpartum infection. Signs and symptoms that merit assessment by the health-care provider include the development of a fever and: Select one: a. Breast engorgement b. Uterine tenderness c. Diarrhea d. Emotional lability

B

To manage her diabetes appropriately and to ensure a good fetal outcome, how would the pregnant woman with diabetes alter her diet? Select one: a. Reduce the carbohydrates in her diet. b. Eat her meals and snacks on a fixed schedule. c. Eat six small equal meals per day. d. Increase her consumption of protein.

B

What family form tends to be the most socially vulnerable? Select one: a. Nuclear family b. Single-parent family c. Extended family d. Married-blended family

B

What is the correct terminology for the nerve block that provides anesthesia to the lower vagina and perineum? Select one: a. Spinal block b. Pudendal c. Epidural d. Local

B

What is the primary nursing responsibility when caring for a client who is experiencing an obstetric hemorrhage associated with uterine atony? Select one: a. Catheterizing the bladder b. Performing fundal massage c. Preparing the woman for surgical intervention d. Establishing venous access

B

What is the primary rationale for nurses wearing gloves when handling the newborn? Select one: a. Because the nurse has the primary responsibility for the baby during the first 2 hours b. To protect the nurse from contamination by the newborn c. As part of the Apgar protocol d. To protect the baby from infection

B

Which STD can be prevented by a vaccine and reduce a person's risk for certain types of cancer? Select one: a. Gonorrhea b. Human papilloma virus c. Hepatitis B d. Herpes simplex virus

B

Which conditions are infants of diabetic mothers (IDMs) at a higher risk for developing? Select one: a. Hyponatremia b. Respiratory distress syndrome c. Iron deficiency anemia d. Sepsis

B

Which information about variations in the infant's blood counts is important for the nurse to explain to the new parents? Select one: a. A somewhat lower-than-expected red blood cell count could be the result of a delay in clamping the umbilical cord. b. An early high white blood cell (WBC) count is normal at birth and should rapidly decrease. c. Platelet counts are higher in the newborn than in adults for the first few months. d. Even a modest vitamin K deficiency means a problem with the blood's ability to properly clot.

B

Which information regarding to injuries to the infant's plexus during labor and birth is most accurate? Select one: a. Parents of children with brachial palsy are taught to pick up the child from under the axillae. b. If the nerves are stretched with no avulsion, then they should completely recover in 3 to 6 months. c. Erb palsy is damage to the lower plexus. d. Breastfeeding is not recommended for infants with facial nerve paralysis until the condition resolves.

B

Which information should the nurse provide to a breastfeeding mother regarding optimal self-care? Select one: a. She will need an extra 1000 calories a day to maintain energy and produce milk. b. She should avoid trying to lose large amounts of weight. c. She must avoid exercising because it is too fatiguing. d. She can return to prepregnancy consumption patterns of any drinks as long as she gets enough calcium.

B

Which instruction should the nurse provide to reduce the risk of nipple trauma? Select one: a. Assess the nipples before each feeding. b. Position the infant so the nipple is far back in the mouth. c. Wash the nipples daily with mild soap and water. d. Limit the feeding time to less than 5 minutes.

B

Which of the following would be a priority for the nurse when caring for a pregnant woman who has recently emigrated from another country? Select one: a. Help her develop a realistic, detailed birth plan. b. Encourage her to include her family for support. c. Teach her about expected emotional changes of pregnancy. d. Refer her to a doula for labor support.

B

Which statement by a newly delivered woman indicates that she knows what to expect regarding her menstrual activity after childbirth? Select one: a. "I will not have a menstrual cycle for 6 months after childbirth." b. "My first menstrual cycle will be heavier than normal and will return to my prepregnant volume within three or four cycles." c. "My first menstrual cycle will be lighter than normal and then will get heavier every month thereafter." d. "My first menstrual cycle will be heavier than normal and then will be light for several months after."

B

Which statement is the best rationale for recommending formula over breastfeeding? Select one: a. Mother lacks confidence in her ability to breastfeed. b. Mother has a medical condition or is taking drugs that could be passed along to the infant via breast milk. c. Other family members or care providers also need to feed the baby. d. Mother sees bottle feeding as more convenient.

B

Which statement related to cephalopelvic disproportion (CPD) is the least accurate? Select one: a. The fetus cannot be born vaginally. b. CPD can be accurately predicted c. Causes of CPD may have maternal or fetal origins. d. CPD can be related to either fetal size or fetal position.

B

Which substance, when abused during pregnancy, is the most significant cause of cognitive impairment and dysfunction in the infant? Select one: a. Tobacco b. Alcohol c. Heroin d. Marijuana

B

Which woman has the highest risk for endometrial cancer? Select one: a. Woman who has been on birth control pills for 15 years b. Postmenopausal woman with hypertension c. Perimenopausal woman who has a cystocele d. Woman who has an intrauterine device (IUD)

B

While obtaining a diet history, the nurse might be told that the expectant mother has cravings for ice chips, cornstarch, and baking soda. Which nutritional problem does this behavior indicate? Select one: a. preeclampsia b. pica c. ptyalism d. purging

B

Your pregnant patient is in her first trimester and is scheduled for an ultrasound. When explaining the rationale for early pregnancy ultrasound, the best response is: Select one: a. "The test will help to determine the baby's position." b. "The test will help to determine how many weeks you are pregnant." c. "The test will help to determine if your baby is growing appropriately." d. "The test will help to determine if you have a boy or girl."

B

The perinatal nurse observes the new mother watching her baby daughter closely, touching her face, and asking many questions about infant feeding. This stage of mothering is best described as: Select one: a. Taking in b. Taking hold c. Taking charge d. Taking time

B As the mother's physical condition improves, she begins to take charge and enters the taking-hold phase where she assumes care for herself and her infant. At this time, the mother eagerly wants information about infant care and shows signs of bonding with her infant. During this phase, the nurse should closely observe mother-infant interactions for signs of poor bonding, and if present, implement actions to facilitate attachment.

The nurse notes that a new father gazes at his baby for prolonged periods of time and comments that his baby is beautiful and he is very happy having a baby. These behaviors are commonly associated with: Select one: a. Bonding b. Engrossment c. Couvade syndrome d. Attachment

B Engrossment is defined as an absorption, preoccupation, and interest shown by fathers with their newborns. New fathers gaze at their newborns for prolonged periods of time as if they are in a hypnotic trance. characteristics of engrossment include a perception that the newborn is perfect.

A primigravida woman at 42 weeks' gestation received Prepidil (dinoprostone) for induction 12 hours ago. The Bishop score is now 3. Which of the following actions by the nurse is appropriate? Select one: a. Perform Nitrazine analysis of the amniotic fluid. b. Report the lack of progress to the obstetrician. c. Place the woman on her left side. d. Ask the provider for an order for oxytocin.

B Prepidil is indicated for cervical ripening, the process of physical softening and opening of the cervix. Cervical status is the most important predictor of successful induction of labor. Cervical status is assessed before induction of labor using the Bishop score. A score of 6 or more is considered favorable for successful induction of labor.

A nurse is performing a newborn assessment on a new admission to the nursery. Which of the following actions should the nurse make when evaluating the baby for congenital dislocation of the hip? Select one: a. Grasp the inner aspects of the baby's calves with thumbs and forefingers. b. Gently abduct the baby's thighs listening for clicks at the joints. c. Palpate the baby's patellae to assess for subluxation of the bones. d. Dorsiflex the baby's feet.

B The Barlow-Ortolani maneuver evaluates for congenital hip dislocation. The infants knees are flexed toward the trunk and thighs are then gently abducted. The examiner observes for symmetrical leg length, full range of motion, no clicks at joints and equal gluteal folds.

Which resource best describes a health care service representing the tertiary level of prevention? Select one: a. Stress prevention seminars b. Premenstrual syndrome (PMS) support group c. Childbirth education classes for single parents d. Self breast exam pamphlet and teaching

B Your answer is correct. The correct answer is: Premenstrual syndrome (PMS) support group

23 y/o Amy delivered a baby 2 months ago and presents to the clinic with complaint of stool coming out of her vagina for the past week. The nurse suspects recto-vaginal fistula. What is the next most appropriate question the nurse should ask Amy? Select one: a. "Are you sure it is stool?" b. "Did you deliver your baby vaginally?" c. "How long has this been going on?" d. "How big was your baby?"

B this question starts the nurse's assessment for potential trauma during childbirth that increased Amy's risk for the fistula.

The perinatal nurse is teaching her new mother about breastfeeding and explains that the most appropriate time to breastfeed is: Select one or more: a. 3 to 4 hours after the last feeding b. When her infant is in a quiet alert state c. When her infant is in an active alert state d. When her infant exhibits hunger-related crying

B & C-When her infant is in a quiet alert state, When her infant is in an active alert state. The optimal time to breastfeed is when the baby is in a quiet alert state. Crying is usually a late sign of hunger, and achieving satisfactory latch-on at this time is difficult. Latch-on is proper attachment of the infant to the breast for feeding. The neonate is most alert during the first 1 to 2 hours after an unmedicated birth, and this is the ideal time to put the infant to the breast.

Susie is 19 y/o and presents to the clinic for her well-woman exam. She denies any problems or family history of cancer. Which of the following are recommended for Susie? (select all that apply). Select one or more: a. Pap smear b. Blood pressure c. Hearing Screen d. HPV test e. Clinical breast exam

B (and only B......)

For which of the following conditions is Daily Fetal Movement Count indicated? (Select all that apply). Select one or more: a. Pregnant woman who is older than 35 years with no complications b. Mother with low amniotic fluid volume or oligohydraminos c. Mother with diabetes during pregnancy d. Mother who complains of headaches during pregnancy that are relieved with Tylenol

B, C

Which of the following nursing interventions are important in the prenatal care of the woman with prenatal depression? (Select all of the following). Select one or more: a. Counsel her on the importance of medication if the woman refuses it. b. Educate the woman about depression and plan of care c. Maintain a caring relationship d. Recommend she see a psychiatrist for management

B, C

A woman seeks care at an infertility clinic. Which of the following tests may this woman undergo to determine what, if any, infertility problem she may have? (Select all that apply.) Select one or more: a. Chorionic villus sampling b. Endometrial biopsy c. Hysterosalpingogram d. Serum FSH analysis

B, C, D

If a woman is at risk for thrombus and is not ready to ambulate, which nursing intervention would the nurse use? (Select all that apply.) Select one or more: a. Having her sit in a chair b. Putting her in antiembolic stockings (thromboembolic deterrent [TED] hose) and/or sequential compression device (SCD) boots c. Having her flex, extend, and rotate her feet, ankles, and legs d. Immediately notifying the physician if a positive Homans sign occurs e. Promoting bed rest

B, C, D

Postoperative nursing care and education for a woman who had an abdominal hysterectomy includes (select all that apply): Select one or more: a. Administering hormone therapy as per provider orders. b. Assessing vaginal bleeding c. Instructing the woman vaginal intercourse until she has followup visit with surgeon. d. Instructing the woman to resume activities as comfort level permits.

B, C, D

The perinatal nurse is teaching the new mother who has chosen to formula feed her infant. Appropriate instructions to be given to this mother include (select all that apply): Select one or more: a. Mix the formula according to manufacturer's instructions; do not overdilute or underdilute b. Periodically check the nipple for slow flow. c. Prepare only enough formula to last for 24 hours and discard open containers or prepared formula after 24 hours. d. Discard any unused formula that remains in a bottle following use.

B, C, D

54 y/o Janice has a history of blood clots in her legs when she used birth control pills. Her hot flushes are "driving me crazy". What alternative comfort measures might be appropriate for Janice? (Select all that apply). Select one or more: a. Take an compound estrogen pill once a week. b. Wear layered clothing c. Avoid triggers that bring about hot flushes d. Avoid heavy blankets at night

B, C, D See Box on Page 139

Typical signs of abusive head trauma (AHT, also known as Shaken Baby Syndrome) include which of the following? (Select all that apply.) Select one or more: a. Broken clavicle b. Poor feeding c. Vomiting d. Breathing problems

B, C, D Symptoms of abusive head trauma are extreme irritability, poor feeding, breathing problems, convulsions, vomiting, and pale or bluish skin.

Which of the following are disadvantages of bottle feeding? (Select all that apply.) Select one or more: a. Increases the frequency of feedings as digestion of formula does not take as long as digestion of breastmilk b. Increases cost compared to breastfeeding c. Increases risk of infection due to lack of maternal antibodies d. Increases risk of childhood obesity

B, C, D Further explination: a. Increases the frequency of feedings as digestion of formula does not take as long as digestion of breastmilk Incorrect Disadvantages of formula feeding include need for increased time to prepare formula; increased cost compared to breastfeeding; increased risk of infection due to lack of antibodies that are naturally present in human milk; increased risk of childhood obesity and insulin-dependent diabetes. One of the advantages of formula feeding is the decrease in frequency of feedings because digestion of formula is slower than that of human milk. b. Increases cost compared to breastfeeding Correct Disadvantages of formula feeding include need for increased time to prepare formula; increased cost compared to breastfeeding; increased risk of infection due to lack of antibodies that are naturally present in human milk; increased risk of childhood obesity and insulin-dependent diabetes. One of the advantages of formula feeding is the decrease in frequency of feedings because digestion of formula is slower than that of human milk. c. Increases risk of infection due to lack of maternal antibodies Correct Disadvantages of formula feeding include need for increased time to prepare formula; increased cost compared to breastfeeding; increased risk of infection due to lack of antibodies that are naturally present in human milk; increased risk of childhood obesity and insulin-dependent diabetes. One of the advantages of formula feeding is the decrease in frequency of feedings because digestion of formula is slower than that of human milk. d. Increases risk of childhood obesity CorrectDisadvantages of formula feeding include need for increased time to prepare formula; increased cost compared to breastfeeding; increased risk of infection due to lack of antibodies that are naturally present in human milk; increased risk of childhood obesity and insulin-dependent diabetes. One of the advantages of formula feeding is the decrease in frequency of feedings because digestion of formula is slower than that of human milk.

Which factors influence cervical dilation? (Select all that apply.) Select one or more: a. Size of the woman b. Scarring of the cervix c. Strong uterine contractions d. Pressure applied by the amniotic sac e. Force of the presenting fetal part against the cervix

B, C, D, E

Cellulitis with or without abscess formation is a fairly common condition. The nurse is providing education for a client whose presentation to the emergency department includes an infection of the breast. Which information should the nurse share with this client? (Select all that apply.) Select one or more: a. Methicillin- resistant Streptococcus aureus is the most common pathogen. b. Treatment for cellulitis or breast infection will include antibiotics. c. Obesity, smoking, and diabetes are risk factors. d. Breast is pale in color and cool to the touch. e. Nipple piercing may be the cause of a recent infection

B, C, E

Which of the following are functions of the placenta? (Select all that apply) Select one or more: a. provides constant thermal environment b. produces hormones, such as progesterone, that helps to relax the uterus c. provides O2 and nutrients to the fetus d. cushions the fetus e. removes CO2 and fetal waste products

B, C, E

While developing an intrapartum care plan for the client in early labor, which psychosocial factors would the nurse recognize upon the client's pain experience? (Select all that apply.) Select one or more: a. Intervention of caregivers b. Anxiety and fear c. Previous experiences with pain d. Support systems e. Culture

B, C, E

16 year old Emily lives in Blackfoot, ID. She has a boyfriend but does not want to have a baby for at least 10 years. She wants an effective, easy birth control method and denies medical problems. She has periods every 4-6 weeks. The nurse would recommend which of the following methods to Emily? (Select all that apply). Select one or more: a. bilateral tubal ligation b. hormonal IUD c. natural family planning d. the implant

B, D

Which of the following is true regarding infertility? (Select all that apply). Select one or more: a. infertility effects approx 5% of reproductive-age couples b. infertility is defined as the inability to achieve pregnancy after a year of unprotected intercourse c. approx 60% of infertility problems are related to the female factors d. anovulation can cause primary and/or secondary infertiity

B, D

Which of the competencies are most relevant to nurses in maternity and women's health? (Select all that apply). Select one or more: a. Refer patient to professional genetic counseling for all of their questions regarding genetics and genomics. b. Collect a thorough family history to include at least 3 generations. c. Provide patient with genomic information without considering their cultural and religious beliefs d. Provide patient with accurate and appropriate genetic and genomic information e. Refer patient to the physician to discussed genetic concerns.

B, D The correct answer is: Collect a thorough family history to include at least 3 generations., Provide patient with accurate and appropriate genetic and genomic information

Supportive care of the infant with neonatal abstinence syndrome (NAS) include both pharmacologic and nonpharmacologic therapy. Nonpharmacologic therapy would include which of the following (select all that apply): Select one or more: a. Clonidine for infants who do not respond to a single agent b. Quiet environment with low lighting and use of soft voices c. Use of oral morphine and methadone d. Swaddling, clustering care, use of pacifiers to promote "self-soothing"

B, D Nonpharmacologic therapy includes quiet environment, low lighting, soft voices, clustering care, swaddling, pacifier use, frequent small feedings, and vertical rocking.

Infants born between 34 0/7 and 36 6/7 weeks of gestation are called late-preterm infants because they have many needs similar to those of preterm infants. Because they are more stable than early-preterm infants, they may receive care that is similar to that of a full-term baby. These infants are at increased risk for which conditions? (Select all that apply.) Select one or more: a. Cardiac distress b. Hyperbilirubinemia c. Hyperglycemia d. Problems with thermoregulation e. Sepsis

B, D, E

A primigravida at 40 weeks of gestation is having uterine contractions every 1½ to 2 minutes and states that they are very painful. Her cervix is dilated 2 cm and has not changed in 3 hours. The woman is crying and wants an epidural. What is the likely status of this woman's labor? Select one: a. She is experiencing a normal latent stage. b. She is exhibiting hypotonic uterine dysfunction. c. She is exhibiting hypertonic uterine dysfunction d. She is experiencing precipitous labor.

C

Nurses play a critical role in educating parents regarding measures to prevent infant abduction. Which instructions contribute to infant safety and security? (Select all that apply.) Select one or more: a. Because of infant security systems, the baby can be left unattended in the client's room. b. The mother should check the photo identification (ID) of any person who comes to her room. c. The baby should be carried in the parent's arms from the room to the nursery. d. Parents should use caution when posting photographs of their infant on the Internet. e. The mom should request that a second staff member verify the identity of any questionable person.

B, D, E

Which of the following lab tests are routinely performed on pregnant women? (Select all that apply). Select one or more: a. Genetic testing for cystic fibrosis b. Syphilis test c. 3 hr GTT d. Blood type e. Rubella titer

B, D, E

The "Period of Purple Crying" is a program developed to educate new parents about infant crying and the dangers of shaking a baby. Each letter in the acronym "PURPLE" represents a key concept of this program. Which concepts are accurate? (Select all that apply.) Select one or more: a. R: baby is resting at last b. U: unexpected c. E: evening d. P: peak of crying and painful expression e. L: extremely loud

B,C, and D- P: peak of crying and painful expression, U: unexpected, E: evening

Which medications are used to manage PPH? (Select all that apply.) Select one or more: a. Terbutaline b. Oxytocin c. Magnesium sulfate d. Hemabate e. Methergine

B,D,E- Oxytocin, Methergine, Hemabate

A woman gave birth to a healthy 7-pound, 13-ounce infant girl. The nurse suggests that the client place the infant to her breast within 15 minutes after birth. The nurse is aware that the initiation of breastfeeding is most effective during the first 30 minutes after birth. What is the correct term for this phase of alertness? Select one: a. Transition period b. Second period of reactivity c. First period of reactivity d. Organizational stage

C

47 y/o Sallie is interested in HRT for her hot flushes and vaginal dryness. She has high blood pressure and her LMP was 3 months ago. She asks the nurse what can she use? What is the nurse's best response? Select one: a. "You cannot take HRT since your blood pressure is high." b. "All women should take the bio-identical estrogen and progesterone pills." c. "It would be best to talk to the health care provider to determine what type of HRT might be best for you." d. "You need to be on the birth control pill for your symptoms and contraception."

C

A NICU nurse is caring for a full-term neonate being treated for group B streptococcus (GBS). The mother of the neonate is crying and shares that she cannot understand how her baby became infected. The best response by the nurse is: Select one: a. "Newborns are more susceptible to infections due to an immature immune system. Would you like additional information on the newborn immune system?" b. "The infection was transmitted to your baby during the birthing process. Do you have a history of sexual transmitted infections?" c. "Approximately 15% to 40% of women have no symptoms but are carriers of group B streptococcus which is found in the vaginal and lower intestinal areas. What other questions do you have regarding your baby's health?" d. "I see that this is very upsetting for you. I will come back later and answer your questions."

C

A client is to take Clomiphene Citrate for infertility. Which of the following is the expected action of this medication? Select one: a. Decrease the symptoms of endometriosis b. Increase serum progesterone levels c. Stimulate release of FSH and LH d. Reduce the acidity of vaginal secretions

C

A diagnostic test commonly used to assess problems of the fallopian tubes is: Select one: a. Endometrial biopsy b. Ovarian reserve testing c. Hysterosalpingogram d. Screening for sexually transmitted infections

C

A first-time dad is concerned that his 3-day-old daughter's skin looks "yellow." In the nurse's explanation of physiologic jaundice, what fact should be included? Select one: a. Physiologic jaundice is also known as breast milk jaundice. b. Physiologic jaundice is caused by blood incompatibilities between the mother and the infant blood types. c. Physiologic jaundice becomes visible when serum bilirubin levels peak between the second and fourth days of life d. Physiologic jaundice occurs during the first 24 hours of life.

C

A healthy 60-year-old African-American woman regularly receives health care at her neighborhood clinic. She is due for a mammogram. At her first visit, her health care provider is concerned about the 3-week wait at the neighborhood clinic and made an appointment for her to have a mammogram at a teaching hospital across town. She did not keep her appointment and returned to the clinic today to have the nurse check her blood pressure. What is the most appropriate statement for the nurse to make to this client? Select one: a. "It's very dangerous to skip your mammograms; your breasts need to be checked." b. "Do you have transportation to the teaching hospital so that you can get your mammogram?" c. "Would you like me to make an appointment for you to have your mammogram here?" d. "I'm concerned that you missed your appointment; let me make another one for you."

C

A multipara, 26 weeks' gestation and accompanied by her husband, has just delivered a fetal demise. Which of the following nursing actions is appropriate at this time? Select one: a. Encourage the parents to pray for the baby's soul. b. Advise the parents that it is better for the baby to have died than to have had to live with a defect. c. Provide opportunities for grieving parents and family members to spend time with the baby d. Advise the parents to refrain from discussing the baby's death with their other children.

C

A neonate born at 36 weeks gestation is classified as which of the following? Select one: a. Very preterm b. Moderately preterm c. Late preterm d. Term

C

A new mother asks whether she should feed her newborn colostrum, because it is not "real milk." What is the nurse's most appropriate answer? Select one: a. Giving colostrum is important in helping the mother learn how to breastfeed before she goes home. b. Colostrum is lower in calories than milk and should be supplemented by formula. c. Colostrum is high in antibodies, protein, vitamins, and minerals. d. Colostrum is unnecessary for newborns.

C

A nurse caring for a family during a loss might notice that a family member is experiencing survivor guilt. Which family member is most likely to exhibit this guilt? Select one: a. Mother b. Siblings c. Grandparents d. Father

C

A patient at 28 weeks' gestation was last seen in the prenatal clinic at 24 weeks' gestation. Which of the following changes should the nurse bring to the attention of the Certified Nurse Midwife? Select one: a. Weight change from 128 pounds to 132 pounds b. Pulse change from 88 bpm to 92 bpm c. Blood pressure change from 110/70 to 140/90 d. Respiratory change from 16 rpm to 20 rpm

C

A postpartum woman, who gave birth 12 hours ago, is breastfeeding her baby. She tells her nurse that she is concerned that her baby is not getting enough food since her milk has not come in. The best response for this patient is: Select one: a. "I understand your concern, but your baby will be okay until your milk comes in." b. "Your baby seems content, so you should not worry about him getting enough to eat." c. "Milk normally comes in around the third day. Prior to that, he is getting colostrum which is high in protein and immunoglobulins which are important for your baby's health." d. "You can bottle feed until your milk comes in."

C

Approximately 8 hours ago, Juanita, a 32-year-old G1 P0, gave birth after 2 hours of pushing. She required an episiotomy and an assisted birth (forceps) due to the weight and size of her baby (9 lb. 9 oz.). The perinatal nurse is performing an assessment of Juanita's perineal area. A slight bulge is palpated and the presence of ecchymoses to the right of the episiotomy is noted. The area feels "full" and is approximately 4 cm in diameter. Juanita describes this area as "very tender." The perinatal nurse notifies the physician of the findings related to Juanita's assessment. The first step in care will most likely be to: Select one: a. Prepare Juanita for surgery b. Administer intravenous fluids c. Apply ice to the perineum d. Insert a urinary catheter

C

Certain changes stimulate chemoreceptors in the aorta and carotid bodies to prepare the fetus for initiating respirations immediately after birth. Which change in fetal physiologic activity is not part of this process? Select one: a. Fetal partial pressure of oxygen (PO2) decreases. b. Fetal lung fluid is cleared from the air passages during labor and vaginal birth. c. Fetal respiratory movements increase during labor. d. Fetal partial pressure of carbon dioxide in arterial blood (PaCO2) increases.

C

During a prenatal visit, the nurse is explaining dietary management to a woman with pregestational diabetes. Which statement by the client reassures the nurse that teaching has been effective? Select one: a. "I will need to eat 600 more calories per day because I am pregnant." b. "I can continue with the same diet as before pregnancy as long as it is well balanced." c. "Diet and insulin needs change during pregnancy." d. "I will plan my diet based on the results of urine glucose testing."

C

How many kilocalories per kilogram (kcal/kg) of body weight does a breastfed term infant require each day? Select one: a. 50 to 65 b. 150 to 200 c. 95 to 110 d. 75 to 90

C

In appraising the growth and development potential of a preterm infant, the nurse should be cognizant of the information that is best described in which statement? Select one: a. Know that the length and breadth of the trunk is the first part of the infant to experience catch-up growth. b. Tell the parents that their child will not catch up until approximately age 10 years (for girls) to age 12 years (for boys). c. Correct for milestones, such as motor competencies and vocalizations, until the child is approximately 2 years of age. d. Know that the greatest catch-up period is between 9 and 15 months postconceptual age.

C

In contrast to placenta previa, what is the most prevalent clinical manifestation of Grade 3 abruptio placentae? Select one: a. Intermittent uterine contractions b. Cramping c. Intense abdominal pain d. Bleeding

C

It is noted that the amniotic fluid of a 42-week gestation baby, born 30 seconds ago, is thick and green and he is not crying vigorously. Which of the following actions by the nurse is critical at this time? Select one: a. Perform a gavage feeding immediately. b. Assess the brachial pulse. c. Assist a physician with intubation and suctioning. d. Stimulate the baby to cry.

C

Mary asks the nurse why does she have to have a speculum exam during her first prenatal visit? The nurse's best answer is: Select one: a. It enables the provider to inspect your vulva and vaginal os. b. It enables the provider to percuss your cervix and uterus c. It enables the provider to inspect your cervix and vaginal walls. d. It allows the provider to palpate the size of your uterus

C

Near the end of the first week of life, an infant who has not been treated for any infection develops a copper-colored maculopapular rash on the palms and around the mouth and anus. The newborn is displaying signs and symptoms of which condition? Select one: a. Herpes simplex virus (HSV) infection b. Gonorrhea c. Congenital syphilis d. HIV

C

Nurses with an understanding of cultural differences regarding likely reactions to pain may be better able to help their clients. Which clients may initially appear very stoic but then become quite vocal as labor progresses until late in labor, when they become more vocal and request pain relief? Select one: a. Arab or Middle Eastern b. Chinese c. Hispanic d. African-American

C

Pelvic floor exercises, also known as Kegel exercises, will help to strengthen the perineal muscles and encourage healing after childbirth. The nurse requests the client to repeat back instructions for this exercise. Which response by the client indicates successful learning? Select one: a. "I contract my thighs, buttocks, and abdomen." b. "I perform 10 of these exercises every day." c. "I pretend that I am trying to stop the flow of urine in midstream." d. "I stand while practicing this new exercise routine."

C

The fetal circulatory structure that connects the pulmonary artery with the descending aorta is known as which of the following? Select one: a. Ductus venosus b. Foramen ovale c. Ductus arteriosus d. Internal iliac artery

C

The labor of a pregnant woman with preeclampsia is going to be induced. Before initiating the oxytocin (Pitocin) infusion, the nurse reviews the woman's latest laboratory test findings, which reveal a platelet count of 90,000 mm3, an elevated aspartate aminotransaminase (AST) level, and a falling hematocrit. The laboratory results are indicative of which condition? Select one: a. Disseminated intravascular coagulation (DIC) syndrome b. Idiopathic thrombocytopenia c. Hemolysis, elevated liver enzyme levels, and low platelet levels (HELLP) syndrome d. Eclampsia

C

The nurse is assessing a full term, quiet, and alert newborn. What is the average expected apical pulse range (in beats per minute)? Select one: a. 100 to 120 b. 150 to 180 c. 120 to 160 d. 80 to 100

C

The nurse is assessing the neonate's skin and notes the presence of a rash with red macules and papules on the trunk. The name for this common neonatal skin condition is: Select one: a. Milia b. Neonatal acne c. Erythema toxicum d. Pustular melanosis

C

The nurse should be cognizant of which condition related to skeletal injuries sustained by a neonate during labor or childbirth? Select one: a. Other than the skull, the most common skeletal injuries are to leg bones. b. Clavicle fractures often need to be set with an inserted pin for stability. c. Unless a blood vessel is involved, linear skull fractures heal without special treatment. d. Newborn's skull is still forming and fractures fairly easily.

C

The nurse should be cognizant of which postpartum physiologic alteration? Select one: a. Lowered white blood cell count after pregnancy can lead to false-positive results on tests for infections. b. Hypercoagulable state protects the new mother from thromboembolism, especially after a cesarean birth. c. Respiratory function returns to nonpregnant levels by 6 to 8 weeks after childbirth d. Cardiac output, pulse rate, and stroke volume all return to prepregnancy normal values within a few hours of childbirth.

C

The nurse takes the history of a client, G2 P1, at her first prenatal visit. The client is referred to a genetic counselor, due to her previous child having a diagnosis of ____. Select one: a. Unilateral amblyopia b. Subdural hematoma c. Sickle cell anemia d. Glomerular nephritis

C

The nurse who provides care to clients in labor must have a thorough understanding of the physiologic processes of maternal hypotension. Which outcome might occur if the interventions for maternal hypotension are inadequate? Select one: a. Early FHR decelerations b. Fetal arrhythmias c. Uteroplacental insufficiency d. Spontaneous rupture of membranes

C

Through a vaginal examination, the nurse determines that a woman is 4 cm dilated. The external fetal monitor shows uterine contractions every 3½ to 4 minutes. The nurse reports this as what stage of labor? Select one: a. Second stage, latent phase b. First stage, transition phase c. First stage, active phase d. First stage, latent phase

C

What is the primary purpose for magnesium sulfate administration for clients with preeclampsia and eclampsia? Select one: a. To shorten the duration of labor b. To prevent a boggy uterus and lessen lochial flow c. To prevent convulsions d. To improve patellar reflexes and increase respiratory efficiency

C

Which condition is considered a medical emergency that requires immediate treatment? Select one: a. Hypotonic uterus b. Uterine atony c. Inversion of the uterus d. ITP

C

Which condition might premature infants who exhibit 5 to 10 seconds of respiratory pauses, followed by 10 to 15 seconds of compensatory rapid respiration, be experiencing? Select one: a. Experiencing severe swings in blood pressure b. Trying to maintain a neutral thermal environment c. Breathing in a respiratory pattern common to premature infants d. Suffering from sleep or wakeful apnea

C

Which description of the phases of the second stage of labor is most accurate? Select one: a. Active phase: Overwhelmingly strong contractions; Ferguson reflux activated; duration of 5 to 15 minutes b. Latent phase: Feeling sleepy; fetal station 2+ to 4+; duration of 30 to 45 minutes c. Descent phase: Significant increase in contractions; Ferguson reflux activated; average duration varies d. Transitional phase: Woman "laboring down"; fetal station 0; duration of 15 minutes

C

Which explanation will assist the parents in their decision on whether they should circumcise their son? Select one: a. Circumcision is rarely painful, and any discomfort can be managed without medication. b. American Academy of Pediatrics (AAP) recommends that all male newborns be routinely circumcised. c. The circumcision procedure has pros and cons during the prenatal period. d. The infant will likely be alert and hungry shortly after the procedure.

C

Which finding would indicate to the nurse that the grieving parents have progressed to the reorganization phase of grieving? Select one: a. The parents have abandoned those moments of "bittersweet grief." b. The parents say that they "feel no pain." c. The parents are discussing sex and a future pregnancy, even if they have not yet sorted out their feelings. d. The parents' questions have progressed from "Why?" to "Why us?"

C

Which is the most accurate description of PPD without psychotic features? Select one: a. Condition that is more common among older Caucasian women because they have higher expectations b. Condition that disappears without outside help c. Distinguishable by irritability d. Postpartum baby blues requiring the woman to visit with a counselor or psychologist

C

Which of the following foods is highest in calcium? Select one: a. An 8 oz. glass of milk b. A 1.5 oz. piece of cheddar cheese c. An 8 oz. container of plain, low-fat yogurt d. A 3 oz. piece of salmon

C

Which of the following statements is true about menopause? Select one: a. Mood changes are unusual in perimenopause b. All women have hot flushes in menopause c. No menses or spotting for 1 year d. Average age of menopause in US women is 56-58 y/o

C

Which of the following tests should never be performed during the first trimester? Select one: a.Maternal screening test for cystic fibrosis b. Noninvasive Prenatal Screening test c. Contraction Stress test d. Ultrasound

C

Which statement best describes chronic hypertension? Select one: a. Chronic hypertension is considered severe when the systolic BP is higher than 140 mm Hg or the diastolic BP is higher than 90 mm Hg. b. Chronic hypertension is defined as hypertension that begins during pregnancy and lasts for the duration of the pregnancy. c. Chronic hypertension can occur independently of or simultaneously with preeclampsia. d. Chronic hypertension is general hypertension plus proteinuria.

C

Which statement is the best rationale for recommending formula over breastfeeding? Select one: a. Mother sees bottle feeding as more convenient. b. Mother lacks confidence in her ability to breastfeed. c. Mother has a medical condition or is taking drugs that could be passed along to the infant via breast milk. d. Other family members or care providers also need to feed the baby.

C

After giving birth to a healthy infant boy, a primiparous client, 16 years of age, is admitted to the postpartum unit. An appropriate nursing diagnosis for her at this time is "Deficient knowledge of infant care." What should the nurse be certain to include in the plan of care as he or she prepares the client for discharge? Select one: a. Advise the client that all mothers instinctively know how to care for their infants. b. Teach the client how to feed and bathe her infant. c. Provide time for the client to bathe her infant after she views a demonstration of infant bathing. d. Give the client written information on bathing her infant.

C Allows nurse to evaluate parental ease with care and adequacy of techniques and provide further instruction if necessary

Which of the following statements indicates that a new mother needs additional teaching? Select one: a. "I need to supervise my cat when she is in the same room as my baby." b. "I will place my baby on her back when she is sleeping." c. "I will not leave my baby on an elevated flat surface after she is able to turn over on her own." d. "I have asked my husband to install safety latches on the lower cabinets."

C Infants should never be left unattended on an elevated flat surface

Folic acid 400mcg a day is recommended to all women of reproductive age for which of the following reasons? Select one: a. reduces a woman's risk for certain cancers b. reduces a woman's risk for osteoporosis and heart disease c. prevents spina bifida and abdominal defects of the baby d. reduces risk for learning disabilities of the baby e. prevents complications during pregnancy

C The correct answer is: prevents spina bifida and abdominal defects of the baby

A married couple lives in a single-family house with their newborn son and the husband's daughter from a previous marriage. Based on this information, what family form best describes this family? Select one: a. Same-sex family b. Extended family c. Married-blended family d. Nuclear family

C Your answer is correct. See pp 18-19 The correct answer is: Married-blended family

Intervention for the sexual abuse survivor is often not attempted by maternity and women's health nurses because of the concern about increasing the distress of the woman and the lack of expertise in counseling. What initial intervention is appropriate and most important in facilitating the woman's care? Select one: a. Setting limits on what the client discloses b. Initiating a referral to an expert counselor c. Listening and encouraging therapeutic communication skills d. Acknowledging the nurse's discomfort to the client as an expression of empathy

C Your answer is correct. The correct answer is: Listening and encouraging therapeutic communication skills

A neonate born at 36 weeks gestation is classified as which of the following? Select one: a. Very preterm b. Moderately preterm c. Late preterm d. Term

C Preterm Births are classified as: Very Premature (<32 weeks gestation); Moderately Premature (32-34 weeks gestation); and Late Premature (34-36 weeks gestation).

A serious but uncommon complication of undiagnosed or partially treated hyperthyroidism is a thyroid storm, which may occur in response to stress such as infection, birth, or surgery. What are the signs and symptoms of this emergency disorder? (Select all that apply). Select one or more: a. Hypertension b. Hypothermia c. Restlessness d. Fever e. Bradycardia

C, D

Which of the following STDs have effective treatment so that a person is no longer infected with the organism? Select all that apply. Select one or more: a. HIV b. human papilloma virus c. syphilis d. chlamydia e. herpes simplex virus

C, D

Which of the following health risks can have a negative impact on a pregnancy? (Select all that apply). Select one or more: a. Negative Family History for Genetic Disorders b. Regular exercise c. Obesity d. Maternal diabetes

C, D

Strategies for culturally responsive care include (select all that apply): Select one or more: a. Practicing ethnocentrism b. Applying stereotyping c. Examining one's own biases d. Learning another language

C, D C:The only actions among the choices that are culturally responsive are examining one's own biases and learning another language or breaking down language barriers. Ethnocentrism and stereotyping are not culturally responsive actions. See pp 26-28 and Box 2-3 D: The only actions among the choices that are culturally responsive are examining one's own biases and learning another language or breaking down language barriers. Ethnocentrism and stereotyping are not culturally responsive actions. See pp 26-28 and Box 2-3

Reports have linked third trimester use of selective serotonin uptake inhibitors (SSRIs) with a constellation of neonatal signs. The nurse is about to perform an assessment on the infant of a mother with a history of a mood disorder. Which signs and symptoms in the neonate may be the result of maternal SSRI use? (Select all that apply.) Select one or more: a. Hyperglycemia b. Hypotonia c. Fever d. Irritability e. Shivering

C, D, E

According to the National Institute of Child Health and Human Development (NICHD) Three-Tier System of FHR Classification, category III tracings include all FHR tracings not categorized as category I or II. Which characteristics of the FHR belong in category III? (Select all that apply.) Select one or more: a. Minimal baseline variability not accompanied by recurrent decelerations b. Variable decelerations without other characteristics such as shoulders or overshoots c. Bradycardia d. Baseline rate of 110 to 160 beats per minute e. Tachycardia f. Absent baseline variability with recurrent variable decelerations

C, E, F

The nurse is performing an assessment on a client who thinks she may be experiencing preterm labor. Which information is the most important for the nurse to understand and share with the client? Select one: a. Braxton Hicks contractions often signal the onset of preterm labor. b. Because all women must be considered at risk for preterm labor and prediction is so variable, teaching pregnant women the symptoms of preterm labor probably causes more harm through false alarms. c. Diagnosis of preterm labor is based on gestational age, uterine activity, and progressive cervical change. d. Because preterm labor is likely to be the start of an extended labor, a woman with symptoms can wait several hours before contacting the primary caregiver.

C. Diagnosis of preterm labor is based on gestational age, uterine activity, and progressive cervical change.

Parents have been asked by the neonatologist to provide breast milk for their newborn son, who was born prematurely at 32 weeks of gestation. The nurse who instructs them regarding pumping, storing, and transporting the milk needs to assess their knowledge of lactation. Which statement is valid? Select one: a. The mother should only pump as much milk as the infant can drink. b. The mother should pump every 2 to 3 hours, including during the night. c. Premature infants more easily digest breast milk than formula. d. A glass of wine just before pumping will help reduce stress and anxiety.

C. Premature infants more easily digest breast milk than formula.

Amy is a 20 year old sexually active woman (G0000) who wants an effective birth control method. She denies any medical problems. Which of the following would the nurse not recommend to Amy? Select one: a. intrauterine device b. the implant c. essure d. oral contraceptive pills

C: Essure

Which statement by the client will assist the nurse in determining whether she is in true labor as opposed to false labor? Select one: a. "I passed some thick, pink mucus when I urinated this morning." b. "My bag of waters just broke." c. "My baby dropped, and I have to urinate more frequently now." d. "The contractions in my uterus are getting stronger and closer together."

D

Which statement by the client would lead the nurse to believe that labor has been established? Select one: a. "My baby dropped, and I have to urinate more frequently now." b. "I passed some thick, pink mucus when I urinated this morning." c. "My bag of waters just broke but I have no pain." d. "The contractions in my uterus are getting stronger and closer together."

D

A 36 y/o pregnant woman has been diagnosed with polyhydraminos. The nurse knows this is based on which of the following? Select one: a. Amniotic fluid index of 10 cm b. Amniotic fluid index of 7 cm c. Amniotic fluid index of 20 cm d. Amniotic fluid index of 30 cm

D

A client is experiencing back labor and complains of intense pain in her lower back. Which measure would best support this woman in labor? Select one: a. Effleurage b. Conscious relaxation or guided imagery c. Pant-blow (breaths and puffs) breathing techniques d. Counterpressure against the sacrum

D

A client with maternal phenylketonuria (PKU) has come to the obstetrical clinic to begin prenatal care. Why would this preexisting condition result in the need for closer monitoring during pregnancy? Select one: a. A pregnant woman is more likely to die without strict dietary control. b. Women with PKU are usually mentally handicapped and should not reproduce. c. PKU is a recognized cause of preterm labor. d. The fetus may develop neurologic problems.

D

A newborn was admitted to the neonatal intensive care unit (NICU) after being delivered at 29 weeks of gestation to a 28-year-old multiparous, married, Caucasian woman whose pregnancy was uncomplicated until the premature rupture of membranes and preterm birth. The newborn's parents arrive for their first visit after the birth. The parents walk toward the bedside but remain approximately 5 feet away from the bed. What is the nurse's most appropriate action? Select one: a. Tell the parents only about the newborn's physical condition and caution them to avoid touching their baby. b. Leave the parents at the bedside while they are visiting so that they have some privacy. c. Wait quietly at the newborn's bedside until the parents come closer. d. Go to the parents, introduce him or herself, and gently encourage them to meet their infant. Explain the equipment first, and then focus on the newborn.

D

A nursery nurse observes that a full-term AGA neonate has nasal congestion, hypertonia, and tremors and is extremely irritable. Based on these observations, the nurse suspects which of the following? Select one: a. Hypoglycemia b. Hypercalcemia c. Cold stress d. Neonatal withdrawal

D

A postpartum client is concerned that her breasts are engorged and uncomfortable. What is the nurse's explanation for this physiologic change? Select one: a. Hyperplasia of mammary tissue b. Overproduction of colostrum c. Accumulation of milk in the lactiferous ducts and glands d. Congestion of veins and lymphatic vessels

D

A pregnant woman at 33 weeks of gestation is brought to the birthing unit after a minor automobile accident. The client is experiencing no pain and no vaginal bleeding, her vital signs are stable, and the FHR is 132 beats per minute with variability. What is the nurse's highest priority? Select one: a. Obtaining a physician's order to discharge her home b. Monitoring the woman for a ruptured spleen c. Monitoring her for 24 hours d. Using continuous EFM for a minimum of 4 hours

D

A pregnant woman has been receiving a magnesium sulfate infusion for treatment of severe preeclampsia for 24 hours. On assessment, the nurse finds the following vital signs: temperature 37.3° C, pulse rate 88 beats per minute, respiratory rate 10 breaths per minute, BP 148/90 mm Hg, absent deep tendon reflexes (DTRs), and no ankle clonus. The client complains, "I'm so thirsty and warm." What is the nurse's immediate action? Select one: a. To administer oxygen b. Call the physician c. To call for an immediate magnesium sulfate level d. To discontinue the magnesium sulfate infusion

D

A premature infant never seems to sleep longer than an hour at a time. Each time a light is turned on, an incubator closes, or people talk near her crib, she wakes up and inconsolably cries until held. What is the correct nursing diagnosis beginning with "ineffective coping, related to"? Select one: a. Physiologic distress b. Behavioral responses c. Severe immaturity d. Environmental stress

D

A premature infant with respiratory distress syndrome (RDS) receives artificial surfactant. How does the nurse explain surfactant therapy to the parents? Select one: a. "The drug keeps your baby from requiring too much sedation." b. "Your baby needs this medication to fight a possible respiratory tract infection." c. "Surfactant is used to reduce episodes of periodic apnea." d. "Surfactant improves the ability of your baby's lungs to exchange oxygen and carbon dioxide."

D

A premature infant with respiratory distress syndrome (RDS) receives artificial surfactant. How does the nurse explain surfactant therapy to the parents? Select one: a. "Surfactant is used to reduce episodes of periodic apnea." b. "Your baby needs this medication to fight a possible respiratory tract infection." c. "The drug keeps your baby from requiring too much sedation." d. "Surfactant improves the ability of your baby's lungs to exchange oxygen and carbon dioxide."

D

A woman arrives at the emergency department with complaints of bleeding and cramping. The initial nursing history is significant for a last menstrual period 6 weeks ago. On sterile speculum examination, the primary care provider finds that the cervix is closed. The anticipated plan of care for this woman would be based on a probable diagnosis of which type of spontaneous abortion? Select one: a. Septic b. Incomplete c. Inevitable d. Threatened

D

A woman at 28 weeks of gestation experiences blunt abdominal trauma as the result of a fall. The nurse must closely observe the client for what? Select one: a. Alteration in maternal vital signs, especially blood pressure b. Complaints of abdominal pain c. Hemorrhage d. Placental abruption

D

A woman gave birth to an infant boy 10 hours ago. Where does the nurse expect to locate this woman's fundus? Select one: a. 2 centimeters below the umbilicus b. Midway between the umbilicus and the symphysis pubis c. Nonpalpable abdominally d. 1 centimeter above the umbilicus

D

A woman who has recently given birth complains of pain and tenderness in her leg. On physical examination, the nurse notices warmth and redness over an enlarged, hardened area. Which condition should the nurse suspect, and how will it be confirmed? Select one: a. von Willebrand disease (vWD); noting whether bleeding times have been extended b. Idiopathic or immune thrombocytopenic purpura (ITP); drawing blood for laboratory analysis c. Disseminated intravascular coagulation (DIC); asking for laboratory tests d. Thrombophlebitis; using real-time and color Doppler ultrasound

D

According to the CDC, all women under the age of 26 years old should be tested for which of the following STDs? Select one: a. Gonorrhea b. HPV c. Zika d. chlamydia

D

Angie is an ER nurse. Her patient complains of a 'painless sore down there' for the last 5 days. The physician diagnoses syphilis and asks Angie to look up the medication to treat the patient. Angie knows the best resource to find this information is: Select one: a. an STD website b. her current drug book c. the 2001 STD book in the ER d. the CDC's STDs Treatment Guidelines 2015

D

Anna wants to know if her baby has Trisomy 18 as early in the pregnancy as possible. She is 8 weeks pregnant. Her physician will suggest which of the following diagnostic test? Select one: a. sequential integrated test b. amniocentesis c. Noninvasive prenatal screening test d. chorionic villi sampling (CVS)

D

Elisa asks the nurse why a bimanual examination is done? Select one: a. It enables the provider to inspect the vulva and vaginal os. b. It allows for the provider to inspect your cervix and vaginal walls. c. It allows for the provider to inspect your uterus and ovaries. d. It allows for the provider to palpate your uterus and ovaries.

D

In appraising the growth and development potential of a preterm infant, the nurse should be cognizant of the information that is best described in which statement? Select one: a. Tell the parents that their child will not catch up until approximately age 10 years (for girls) to age 12 years (for boys). b. Know that the length and breadth of the trunk is the first part of the infant to experience catch-up growth. c. Know that the greatest catch-up period is between 9 and 15 months postconceptual age. d. Correct for milestones, such as motor competencies and vocalizations, until the child is approximately 2 years of age.

D

In follow-up appointments or visits with parents and their new baby, it is useful if the nurse can identify infant behaviors that can either facilitate or inhibit attachment. What is an inhibiting behavior? Select one: a. The infant's activity is somewhat predictable. b. The infant cries only when hungry or wet. c. The infant clings to the parents. d. The infant seeks attention from any adult in the room.

D

Jenny, a 21-year-old single woman, comes for her first prenatal appointment at 31 weeks' gestation with her first pregnancy. The clinic nurse's most appropriate statement is: Select one: a. "Jenny, it is late in your pregnancy to be having your first appointment, but it is nice to meet you and I will try to help you get caught up in your care." b. "Jenny, have you had care in another clinic? I can't believe this is your first appointment!" c. "Jenny, by the date of your last menstrual period, you are 31 weeks and now that you are finally here, we need you to come monthly for the next two visits and then weekly." d. "Jenny, by your information, you are 31 weeks' gestation in this pregnancy. Do you have questions for me before I begin your prenatal history and information sharing?"

D

Many clients are concerned about the increased levels of mercury in fish and may be reluctant to include this source of nutrients in their diet. What is the best advice for the nurse to provide? Select one: a. Canned white tuna is a preferred choice. b. Fish caught in local waterways is the safest. c. Salmon and shrimp contain high levels of mercury. d. Shark, swordfish, and mackerel should be avoided.

D

Near the end of the first week of life, an infant who has not been treated for any infection develops a copper-colored maculopapular rash on the palms and around the mouth and anus. The newborn is displaying signs and symptoms of which condition? Select one: a. Gonorrhea b. HIV c. Herpes simplex virus (HSV) infection d. Congenital syphilis

D

Sally is 34 weeks pregnant and has a biophysical profile score of 8. She asks the nurse what does that mean? The nurse's best response is: Select one: a. "Your baby is in danger of dying and you will probably need to deliver ASAP. I will call your doctor." b. " Your baby may be having some problems and your doctor may want to check to see if it is safe to deliver her soon. I will call your doctor." c. "Since you are more than 36 weeks, it is best to deliver your baby as she may have asphyxia. I will call your doctor." d. "Your baby has low risk for having a problem having enough oxygen in her body and your doctor will probably want to do the test again soon. I will call your doctor. "

D

The clinic nurse sees Xiao and her infant in the clinic for their 2-week follow-up visit. Xiao appears to be tired, her clothes and hair appear unwashed, and she does not make eye contact with her infant. She is carrying her son in the infant carrier and when asked to put him on the examining table, she holds him away from her body. The clinic nurse's most appropriate question to ask would be: Select one: a. "What has happened to you?" b. "Do you have help at home?" c. "Is there anything wrong with your son?" d. "Would you tell me about the first few days at home?"

D

The nurse has received a report regarding a client in labor. The woman's last vaginal examination was recorded as 3 cm, 30%, and -2. What is the nurse's interpretation of this assessment? Select one: a. Cervix is effaced 3 cm and dilated 30%; the presenting part is 2 cm below the ischial spines. b. Cervix is effaced 3 cm and dilated 30%; the presenting part is 2 cm above the ischial spines. c. Cervix is dilated 3 cm and effaced 30%; the presenting part is 2 cm below the ischial spines. d. Cervix is dilated 3 cm and effaced 30%; the presenting part is 2 cm above the ischial spines.

D

The nurse is preparing to administer methotrexate to the client. This drug is most often used for which obstetric complication? Select one: a. Missed abortion b. Complete hydatidiform mole c. Abruptio placentae d. Unruptured ectopic pregnancy

D

The nurse is teaching new parents about metabolic screening for the newborn. Which statement is most helpful to these clients? Select one: a. All states test for phenylketonuria (PKU), hypothyroidism, cystic fibrosis, and sickle cell diseases. b. Federal law prohibits newborn genetic testing without parental consent. c. Hearing screening is now mandated by federal law. d. If genetic screening is performed before the infant is 24 hours old, then it should be repeated at age 1 to 2 weeks.

D

The nurse who is caring for a woman hospitalized for hyperemesis gravidarum would expect the initial treatment to involve what? Select one: a. Enteral nutrition to correct nutritional deficits b. Corticosteroids to reduce inflammation c. Antiemetic medication, such as pyridoxine, to control nausea and vomiting d. Intravenous (IV) therapy to correct fluid and electrolyte imbalances

D

The perinatal nurse realizes that an FHR that is tachycardic, bradycardic, has late decelerations, or loss of variability is nonreassuring and is associated with which condition? Select one: a. Hypotension b. Maternal drug use c. Cord compression d. Hypoxemia

D

Under which circumstance should the nurse assist the laboring woman into a hands-and-knees position? Select one: a. Membranes have ruptured. b. Fetus is in a vertex presentation. c. Fetus is at or above the ischial spines. d. Occiput of the fetus is in a posterior position

D

What important, immediate postoperative care practice should the nurse remember when caring for a woman who has had a mastectomy? Select one: a. The affected arm should be held down close to the woman's side. b. The affected arm should be used for intravenous (IV) therapy. c. Venipuncture for blood work should be performed on the affected arm. d. The blood pressure (BP) cuff should not be applied to the affected arm.

D

What information is important for the nurse to include in planning for the care of a woman who has had a vaginal hysterectomy (no oopherectomy) for fibroids? Select one: a. Next pap smear is due in one year. b. Expect surgical menopause. c. Expect to be fully recovered in 4 to 6 weeks. d. Expect no changes in her hormone levels.

D

What information regarding a fractured clavicle is most important for the nurse to take into consideration when planning the infant's care? Select one: a. Prone positioning facilitates bone alignment. b. The shoulder should be immobilized with a splint. c. Parents should be taught range-of-motion exercises. d. No special treatment is necessary.

D

What is one of the initial signs and symptoms of puerperal infection in the postpartum client? Select one: a. Fatigue continuing for longer than 1 week b. Profuse vaginal lochia with ambulation c. Pain with voiding d. Temperature of 38° C (100.4° F) or higher on 2 successive days

D

When a nulliparous woman telephones the hospital to report that she is in labor, what guidance should the nurse provide or information should the nurse obtain? Select one: a. Arrange for the woman to come to the hospital for labor evaluation. b. Emphasize that food and fluid intake should stop. c. Tell the woman to stay home until her membranes rupture. d. Ask the woman to describe why she believes she is in labor.

D

Which client is most likely to experience strong and uncomfortable afterpains? Select one: a. A woman whose infant weighed 5 pounds, 3 ounces b. A woman who is bottle-feeding her infant c. A woman who experienced oligohydramnios d. A woman who is a gravida 4, para 4-0-0-4

D

Which clinical findings would alert the nurse that the neonate is expressing pain? Select one: a. Cry face; flaccid limbs; closed mouth b. Low-pitched crying; tachycardia; eyelids open wide c. High-pitched, shrill cry; withdrawal; change in heart rate d. Cry face; eyes squeezed; increase in blood pressure

D

Which conditions are infants of diabetic mothers (IDMs) at a higher risk for developing? Select one: a. Sepsis b. Hyponatremia c. Iron deficiency anemia d. Respiratory distress syndrome

D

Which description of the four stages of labor is correct for both the definition and the duration? Select one: a. Fourth stage: delivery of the placenta to recovery; 30 minutes to 1 hour b. Third stage: active pushing to birth; 20 minutes (multiparous woman), 50 minutes (nulliparous woman) c. Second stage: full effacement to 4 to 5 cm; visible presenting part; 1 to 2 hours d. First stage: onset of regular uterine contractions to full dilation; less than 1 hour to 20 hours

D

Which instruction should the nurse provide to reduce the risk of nipple trauma? Select one: a. Assess the nipples before each feeding. b. Wash the nipples daily with mild soap and water. c. Limit the feeding time to less than 5 minutes. d. Position the infant so the nipple is far back in the mouth.

D

Which of the following cancers has no accurate screening test for secondary prevention? Select one: a. Cervical cancer b. Breast cancer c. Uterine cancer d. Ovarian cancer

D

Which of the following vaginitis conditions is a sexually transmitted infection? Select one: a. monolial vaginitis b. atrophic vaginitis c. bacterial vaginosis d. trichomonas vaginitis

D

Which options for saying "good-bye" would the nurse want to discuss with a woman who is diagnosed with having a stillborn girl? Select one: a. The nurse should not discuss any options at this time; plenty of time will be available after the baby is born. b. "Would you like a picture taken of your baby after birth?" c. "What funeral home do you want notified after the baby is born?" d. "When your baby is born, would you like to see and hold her?"

D

While discussing the societal impacts of breastfeeding, the nurse should be cognizant of the benefits and educate the client accordingly. Which statement as part of this discussion would be incorrect? Select one: a. Breastfeeding benefits the environment. b. Breastfeeding saves families money. c. Breastfeeding requires fewer supplies and less cumbersome equipment. d. Breastfeeding costs employers in terms of time lost from work.

D

The perinatal nurse listens as Chantal describes her labor and emergency cesarean birth. Providing an opportunity to review this experience may assist Chantal in: Select one: a. Her role development in the "letting go" stage b. Decreasing her ambivalence about her labor and birth c. Understanding her guilt involved in her labor and birth d. Developing more positive feelings about her labor and birth

D After a cesarean birth, especially when unplanned, nurses must be aware of the myriad of potential psychological issues that may arise. Research suggests that women may perceive cesarean birth to be a less positive experience than a vaginal birth. Unplanned or emergent cesarean deliveries and the experience of cesarean birth may be associated with more negative perceptions of the birthing experience. Allowing Chantal to talk about the experience can help her develop a more positive attitude about her own experience.

If the umbilical cord prolapses during labor, the nurse should immediately: Select one: a. Type and cross-match blood for an emergency transfusion. b. Await provider order for preparation for an emergency cesarean section. c. Attempt to reposition the cord above the presenting part. d. Perform vaginal exam and lifting the presenting part off of the cord to relieve pressure on the cord.

D Prolapse of the umbilical cord is when the cord lies below the presenting part of the fetus. The cord becomes trapped against the presenting part and circulation is occluded; FHR will usually show bradycardia or prolonged decel. An emergency cesarean birth is typically performed. Occlusion of the cord may be partially relieved by lifting the presenting part off the cord with a vaginal exam. The examiner's hand remains in the vagina, lifting the presenting part off the cord until delivery by cesarean. There is no attempt to push the cord above the presenting part. Type and screen of blood is generally done on admission for all laboring women; type and cross-match can readily be accomplished using the blood sample already in the lab.

The perinatal nurse explains to the student nurse that ____ is the leading cause of neonatal death in the United States. Select one: a. Sudden Infant Death Syndrome b. Respiratory distress of newborns c. Effects of maternal complications d. Congenital malformations

D See page 6 The correct answer is: Congenital malformations The leading causes of neonatal deaths (in order) are: 1) congenital malformations; 2) disorders related to short gestation and low birth weight; and 3) sudden infant death syndrome.

Documentation of abuse can be useful to women later in court, should they elect to press charges. It is of key importance for the nurse to document accurately at the time that the client is seen. Which entry into the medical record would be the least helpful to the court? Select one: a. Photographs of injuries b. Accurate description of the client's demeanor c. Clear and legible written documentation d. Summary of information (e.g., "The client is a battered woman.")

D See pp 107-109 Your answer is correct. The correct answer is: Summary of information (e.g., "The client is a battered woman.")

The perinatal nurse explains to the student nurse that a goal of the Healthy People 2020 is to: Select one: a. Increase proportion of infants who are bottle-feed. b. Increase number of unintended pregnancies. c. Increase the number of babies who are born preterm. d. Increase proportion of infants who are breastfed.

D The correct answer is: Increase proportion of infants who are breastfed.

A woman gave birth to a 7-pound, 6-ounce infant girl 1 hour ago. The birth was vaginal and the estimated blood loss (EBL) was 1500 ml. When evaluating the woman's vital signs, which finding would be of greatest concern to the nurse? Select one: a. Temperature 36.8° C, heart rate 60 bpm, respirations 18 breaths per minute, and blood pressure 140/90 mm Hg b. Temperature 37.4° C, heart rate 88 bpm, respirations 36 breaths per minute, and blood pressure 126/68 mm Hg c. Temperature 38° C, heart rate 80 bpm, respirations 16 breaths per minute, and blood pressure 110/80 mm Hg d. Temperature 37.9° C, heart rate 120 beats per minute (bpm), respirations 20 breaths per minute, and blood pressure 90/50 mm Hg

D These vital signs best demonstrate the body's attempt to compensate for significant blood loss

To provide culturally competent care to an Asian-American family, which question should the nurse include during the assessment interview? Select one: a. "Do you want some milk to drink?" b. "Do you want music playing while you are in labor?" c. "Do you have a name selected for the baby?" d. "Do you prefer hot or cold beverages?"

D Your answer is correct. See Table 2-2, pp 26-27 The correct answer is: "Do you prefer hot or cold beverages?"

A health care provider performs a clinical breast examination on a woman diagnosed with fibroadenoma. How would the nurse explain the defining characteristics of a fibroadenoma? (Select all that apply). Select one or more: a. Inflammation of the milk ducts and glands behind the nipples b. Thick, sticky discharge from the nipple of the affected breast c. Lumpiness in both breasts that develops 1 week before menstruation d. Single lump in one breast that can increase in size. e. Does not increase risk for breast cancer

D, E

Felicity Chan, a new mother, is accompanied by her mother during her hospital stay on the postpartum unit. Felicity's mother makes specific, various requests of the nurses including bringing warm tea, a cot to sleep on, and that the baby not be bathed at this time. Felicity's mother is also concerned about the amount of work that Felicity may be doing in the provision of infant care. Felicity asks for help with breastfeeding. After Felicity has finished breastfeeding, her mother asks for a bottle so they can warm it and "feed" the baby. How would the perinatal nurse best respond to Felicity's mother in a culturally sensitive way? Select one: a. Ask Felicity's mother to leave for 30 minutes to allow for some private time with Felicity to explore her learning needs privately. b. Ask both Felicity and her mother about the preferred infant feeding method, and assess what they already know. c. Convey to Felicity and her mother an understanding of the concepts of "hot" and "cold" within their belief system. d. Ask Felicity what she knows about breastfeeding, and provide information to both women to support Felicity's decision.

D. Ask Felicity what she knows about breastfeeding, and provide information to both women to support Felicity's decision.

A patient, G1 P0, is admitted to the labor and delivery unit for induction of labor. The following assessments were made on admission: Bishop score of 4, fetal heart rate 140s with moderate variability and no decelerations, TPR 98.6°F, 88, 20, BP 120/80, negative obstetrical history. A prostaglandin suppository was inserted at that time. Which of the following findings, 6 hours after insertion, would warrant the removal of the Cervidil (dinoprostone)? Select one: a. Bishop score of 5 b. Fetal heart of 152 bpm c. Respiratory rate of 24 rpm d. More than 5 contractions in 10 minutes

D. More than 5 contractions in 10 minutes. This is hyperstimulation

The nurse who elects to work in the specialty of obstetric care must have the ability to distinguish between preterm birth, preterm labor, and low birth weight. Which statement regarding this terminology is correct? Select one: a. Low birth weight is a newborn who weighs below 3.7 pounds. b. Terms preterm birth and low birth weight can be used interchangeably. c. Preterm birth rate in the United States continues to increase. d. Preterm labor is defined as cervical changes and uterine contractions occurring between 20 and 37 weeks of gestation.

D. Preterm labor is defined as cervical changes and uterine contractions occurring between 20 and 37 weeks of gestation.

What is the primary purpose for the use of tocolytic therapy to suppress uterine activity? Select one: a. If the client develops pulmonary edema while receiving tocolytic therapy, then intravenous (IV) fluids should be given. b. Drugs can be efficaciously administered up to the designated beginning of term at 37 weeks gestation. c. Tocolytic therapy has no important maternal (as opposed to fetal) contraindications. d. The most important function of tocolytic therapy is to provide the opportunity to administer antenatal glucocorticoids

D. The most important function of tocolytic therapy is to provide the opportunity to administer antenatal glucocorticoids

Allison is 32 weeks pregnant presents to L&D and tells the nurse she has not felt her baby move in 48 hours. The nurse begins electronic fetal monitoring and notes the baby's heart rate is in the 130's. The nurse contacts Allison's physician to obtain an order for which of the following tests? Select one: a. Leopolds Maneuvers b. Fetal fibronectin c. Maternal Serum Screening Test d. Biophysical profile e. Non stress Test

E

Which of the following is the most effective contraception? Select one: a. condoms b. oral contraceptive pills c. natural family planning d. the vaginal ring e. the implant

E

While taking a family history, 37 y/o Bernice tells the nurse her paternal grandmother had breast and colon cancer in her 60's, two paternal aunts had breast cancer in their 40's, one of those aunts also had ovarian cancer in her 40's, one paternal uncle had colon cancer in his 40's, her paternal cousin has breast cancer in her 40's, and her father had colon cancer in his 40's. She denies cancer on her mother's side of the family. Bernice states she has 2 younger sisters. Which of the following responses would be best for the nurse to say to Bernice about her reported family history? Select one: a. "Have you had your pap smear recently?" b. "Do you have cancer on your mother's side of the family?" c. "Have you had a screening mammogram?" d. "Since all of these cancers are on your father's side of the family, you have no increased risk for cancer." e. "The types of cancers reported on your father's side of the family may be due to a gene mutation that can be passed onto to each generation. Have any of your relatives or you considered genetic testing?"

E

The clinic nurse recognizes that the longer an infant is formula fed, the greater is the immunity and resistance the infant will develop against bacterial and viral infections. Select one: True False

False

The postpartum nurse is caring for a couple who experienced an unplanned emergency cesarean birth. The nurse observes the following behaviors: Parents are gently touching their newborn. Mother is softly singing to her baby. Father is gazing into his baby's eyes. Based on this data, the correct nursing diagnosis is altered parent-infant bonding related to emergency cesarean birth. Select one: True False

False

The vaginal examination is an essential component of labor assessment. It reveals whether the client is in true labor and enables the examiner to determine whether membranes have ruptured. The vaginal examination is often stressful and uncomfortable for the client and should be performed only when indicated. Match the correct step number, from 1 to 7, with each component of a vaginal examination of the laboring woman. A: Cleanse the perineum and vulva, if necessary. B: After obtaining permission, gently insert the index and middle fingers into the vagina C: Position the woman to prevent supine hypotension. D: Document findings and report to the health care provider. E: Determine dilation, presenting part, status of membranes, and characteristics of amniotic fluid. F: Explain the findings to the client. G: Use sterile gloves and soluble gel for lubrication.

G, C, A, B, E, F, D Use sterile gloves and soluble gel for lubrication. Position the woman to prevent supine hypotension. Cleanse the perineum and vulva, if necessary. After obtaining permission, gently insert the index and middle fingers into the vagina Determine dilation, presenting part, status of membranes, and characteristics of amniotic fluid. Explain the findings to the client. Document findings and report to the health care provider.

The nurse assessing a newborn for heat loss is aware that nonshivering thermogenesis utilizes the newborn's stores of brown adipose tissue (BAT) to provide heat in the cold-stressed newborn. Select one: True False

True Brown adipose tissue, also known as "brown fat," is a unique highly vascular fat found only in newborns. BAT promotes an increase in metabolism, heat production, heat transfer to the peripheral system. Heat is produced by intense lipid metabolic metabolism but reserves are rapidly depleted during periods of cold stress.

Which woman is at the greatest risk for psychologic complications after hysterectomy? Select one: a. 19-year-old woman who had a ruptured uterus after giving birth to her first child b. 55-year-old woman who has been having abnormal bleeding and pain for 3 years c. 62-year-old widow who has three friends who have had uncomplicated hysterectomies d. 46-year-old woman who has had three children and has just been promoted at work

a. 19-year-old woman who had a ruptured uterus after giving birth to her first child

Which options for saying "good-bye" would the nurse want to discuss with a woman who is diagnosed with having a stillborn girl? Select one: a. "When your baby is born, would you like to see and hold her?" ] b. "What funeral home do you want notified after the baby is born?" c. The nurse should not discuss any options at this time; plenty of time will be available after the baby is born. d. "Would you like a picture taken of your baby after birth?"

a. "When your baby is born, would you like to see and hold her?"

A client is warm and asks for a fan in her room for her comfort. The nurse enters the room to assess the mother and her infant and finds the infant unwrapped in his crib with the fan blowing over him on high. The nurse instructs the mother that the fan should not be directed toward the newborn and that the newborn should be wrapped in a blanket. The mother asks why. How would the nurse respond? Select one: a. "Your baby may lose heat by convection, which means that he will lose heat from his body to the cooler ambient air. You should keep him wrapped, and should prevent cool air from blowing on him." b. "Your baby may lose heat by evaporation, which means that he will lose heat from his body to the cooler ambient air. You should keep him wrapped, and should prevent cool air from blowing on him." c. "Your baby may lose heat by conduction, which means that he will lose heat from his body to the cooler ambient air. You should keep him wrapped, and should prevent cool air from blowing on him." d. "Your baby will easily get cold stressed and needs to be bundled up at all times."

a. "Your baby may lose heat by convection, which means that he will lose heat from his body to the cooler ambient air. You should keep him wrapped, and should prevent cool air from blowing on him." CorrectBest answer!

Which of the following is an indication for the administration of methylergonovine (Methergine)? Select one: a. Boggy uterus that does not respond to massage and oxytocin therapy b. Woman with a large hematoma c. Woman with a deep vein thrombosis d. Woman with severe postpartum depression

a. Boggy uterus that does not respond to massage and oxytocin therapy. Methergine is administered IM or IV in the presence of postpartum hemorrhage due to uterine atony or subinvolution when fundal massage and oxytocin therapy are ineffective. Always check BP before administration and notify provider before injection if elevated as Methergine can increase BP.

A postpartum woman has been diagnosed with postpartum psychosis and will shortly be admitted to the psychiatric unit. Which of the following actions should the nurse perform to ensure safety for both mother and infant? Select one: a. Closely monitor all mother-infant interactions. b. Maintain client on strict bed rest. c. Restrict visitation to her partner. d. Carefully monitor toileting.

a. Closely monitor all mother-infant interactions. Postpartum psychosis (PPP) is a variant of bipolar disorder and is the most serious form of postpartum mood disorders. Onset of symptoms can be as early as the 3rd postpartum day. Assessment findings include paranoia, delusions associated with the baby, mood swings, extreme agitation, confused thinking, and strange beliefs.

Health promotion includes contraception for which of the following reasons? Select one: a. Contraception can positively affect future health. b. Unplanned pregnancies are always welcomed. c. Child spacing and planned pregnancies do not improve perinatal outcomes, d. Achieving desired family size does not enable a better sharing of resources.

a. Contraception can positively affect future health. Correct

The clinic nurse uses Leopold maneuvers to determine the fetal lie, presentation, and position. The nurse's hands are placed on the maternal abdomen to gently palpate the fundal region of the uterus. This action is best described as the: Select one: a. First maneuver b. Second maneuver c. Third maneuver d. Fourth maneuver

a. First maneuver Correct

Which collection of risk factors will most likely result in damaging lacerations, including episiotomies? Select one: a. First-time mother with reddish hair whose rapid labor was overseen by an obstetrician b. Dark-skinned first-time mother who is going through a long labor c. Dark-skinned woman who has had more than one pregnancy, who is going through prolonged second-stage labor, and who is attended by a midwife d. Reddish-haired mother of two who is going through a breech birth

a. First-time mother with reddish hair whose rapid labor was overseen by an obstetrician

The nurse is using the New Ballard Scale to determine the gestational age of a newborn. Which assessment finding is consistent with a gestational age of 40 weeks? Select one: a. Flexed posture b. Abundant lanugo c. Smooth, pink skin with visible veins d. Faint red marks on the soles of the feet

a. Flexed posture

Which cardiovascular changes cause the foramen ovale to close at birth? Select one: a. Increased pressure in the left atrium b. Changes in the hepatic blood flow c. Decreased blood flow to the left ventricle d. Increased pressure in the right atrium

a. Increased pressure in the left atrium

A woman gave birth to a healthy infant boy 5 days ago. What type of lochia does the nurse expect to find when evaluating this client? Select one: a. Lochia serosa b. Lochia sangra c. Lochia rubra d. Lochia alba

a. Lochia serosa CorrectCorrect!

The perinatal nurse observed the pediatrician completing the Ballard Maturational Score (BMS). The maturity components used with this assessment are (select all that apply): Select one or more: a. Physical b. Behavioral c. Reflexive d. Neuromuscular

a. Physical d. Neuromuscular Correct - With the BMS, the infant examination yields a score of neuromuscular and physical maturity that can be extrapolated onto a corresponding age scale to reveal the infant's gestational age in weeks.

Tanya, a 30-year-old woman, is being prepared for a planned cesarean birth. The perinatal nurse assists the anesthesiologist with the spinal block and then positions Tanya in a supine position. Tanya's blood pressure drops to 90/52, and there is a decrease in the fetal heart rate to 110 bpm. The perinatal nurse's best response is to: Select one: a. Place Tanya in a left lateral tilt b. Discontinue Tanya's intravenous administration. c. Have naloxone (Narcan) ready for administration. d. Have epinephrine ready for administration.

a. Place Tanya in a left lateral tilt Correct - Reposition the woman after epidural or spinal anesthesia in a supine position with a left lateral tilt to decrease the pressure from the uterus on the inferior vena cava and to maintain placental perfusion.

While providing care to the maternity client, the nurse should be aware that one of these anxiety disorders is likely to be triggered by the process of labor and birth. Which disorder fits this criterion? Select one: a. Posttraumatic stress disorder (PTSD) b. Phobias c. Obsessive-compulsive disorder (OCD) d. Panic disorder

a. Posttraumatic stress disorder (PTSD) CorrectCorrect

Which of the following is true about twin pregnancy? Select one: a. Pregnant women with twins usually require more surveillance during their pregnancy compared to women with singleton pregnancy. b. There is no more risk of complications during a twin pregnancy as compared to a singleton pregnancy. c. The cesarean section rate for women with a singleton pregnancy is more than that for women with a twin pregnancy. d. Pregnant women with twins should gain weight as recommended for women with a singleton pregnancy.

a. Pregnant women with twins usually require more surveillance during their pregnancy compared to women with singleton pregnancy. CorrectCorrect!

A pregnant woman who has a history of cesarean births is requesting to have a vaginal birth after cesarean (VBAC). In which of the following situations should the nurse advise the patient that her request may be declined? Select one: a. Previous uterine surgery b. Flexed fetal attitude c. Previous low flap uterine incision d. Positive vaginal candidiasis

a. Previous uterine surgery

What is the primary rationale for the thorough drying of the infant immediately after birth? Select one: a. Reduces heat loss from evaporation b. Increases blood supply to the hands and feet c. Stimulates crying and lung expansion d. Removes maternal blood from the skin surface

a. Reduces heat loss from evaporation CorrectCorrect! Prevents cold stress which can cause problems for the neonate.

An example of a cultural prescriptive belief during pregnancy is: Select one: a. Remain active during pregnancy b. Coldness in any form should be avoided c. Do not have your picture taken d. Avoid sexual intercourse during the third trimester

a. Remain active during pregnancy Correct The belief that the patient should remain active during pregnancy is the only example of a cultural prescriptive belief. All of the other answers are examples of cultural restrictive or proscriptive beliefs. see page 331

In evaluating the effectiveness of magnesium sulfate for the treatment of preterm labor, which finding alerts the nurse to possible side effects? Select one: a. Serum magnesium level of 10 mg/dl b. Respiratory rate (RR) of 16 breaths per minute c. Urine output of 160 ml in 4 hours d. DTRs 2+ and no clonus

a. Serum magnesium level of 10 mg/dl Correct

A pregnant woman at term is transported to the emergency department (ED) after a severe vehicular accident. The obstetric nurse responds and rushes to the ED with a fetal monitor. Cardiopulmonary arrest occurs as the obstetric nurse arrives. What is the highest priority for the trauma team? Select one: a. Starting cardiopulmonary resuscitation (CPR) b. Transferring the woman to the surgical unit for an emergency cesarean delivery in case the fetus is still alive c. Obtaining IV access, and starting aggressive fluid resuscitation d. Quickly applying the fetal monitor to determine whether the fetus viability

a. Starting cardiopulmonary resuscitation (CPR)

A gravida, G4 P1203, fetal heart rate 150s, is 14 weeks pregnant, fundal height 1 cm above the symphysis. She denies experiencing quickening. Which of the following nursing conclusions made by the nurse is correct? Select one: a. The woman is experiencing a normal pregnancy. b. The woman may be having difficulty accepting this pregnancy. c. The woman must see a nutritionist as soon as possible. d. The woman will likely miscarry the conceptus.

a. The woman is experiencing a normal pregnancy. Correct

The nurse is circulating during a cesarean birth of a preterm infant. The obstetrician requests that cord clamping be delayed. What is the rationale for this directive? Select one: a. To reduce the risk of intraventricular hemorrhage b. To improve the ability to fight infection c. To decrease total blood volume d. To reduce the risk for jaundice

a. To reduce the risk of intraventricular hemorrhage

A client at 39 weeks of gestation has been admitted for an external version. Which intervention would the nurse anticipate the provider to order? Select one: a. Tocolytic drug b. Contraction stress test (CST) c. Local anesthetic d. Foley catheter

a. Tocolytic drug CorrectCorrect!

A parent who has a hearing impairment is presented with a number of challenges in parenting. Which nursing approaches are appropriate for working with hearing-impaired new parents? (Select all that apply.) Select one or more: a. Using devices that transform sound into light b. Speaking quickly and loudly c. Assuming that the client knows sign language d. Writing messages that aid in communication e. Ascertaining whether the client can read lips before teaching

a. Using devices that transform sound into light Correct d. Writing messages that aid in communication Correct e. Ascertaining whether the client can read lips before teaching Correct

What information should the nurse understand fully regarding rubella and Rh status? Select one: a. Women should be warned that the rubella vaccination is teratogenic and that they must avoid pregnancy for at least 1 month after vaccination b. Breastfeeding mothers cannot be vaccinated with the live attenuated rubella virus. c. Rh immunoglobulin is safely administered intravenously because it cannot harm a nursing infant. d. Rh immunoglobulin boosts the immune system and thereby enhances the effectiveness of vaccinations.

a. Women should be warned that the rubella vaccination is teratogenic and that they must avoid pregnancy for at least 1 month after vaccination

For which of the following conditions is colposcopy used to further evaluate need for intervention? (Select all that apply). Select one or more: a. cervical cancer b. uterine cancer c. bacterial vaginosis d. vaginal cancer e. uterine fibroids

a. cervical cancer d. vaginal cancer

A woman has preinvasive cancer of the cervix. Which modality would the nurse discuss as an available option for a client with this condition? Select one: a. cryosurgery b. hysterectomy c. internal radiation d. colposcopy

a. cryosurgery

What is the risk for all pregnancies to have a fetus with minor birth defects? Select one: a. 5-9% b. 10-15% c. 3-4% d. Less than 1%

b. 10-15%

Which statement regarding the postpartum uterus is correct? Select one: a. At the end of the third stage of labor, the postpartum uterus weighs approximately 500 g. b. After 2 weeks postpartum, it should be abdominally nonpalpable. c. After 2 weeks postpartum, it weighs 100 g. d. Postpartum uterus returns to its original (prepregnancy) size by 6 weeks postpartum

b. After 2 weeks postpartum, it should be abdominally nonpalpable

A pregnant woman arrives at the birth unit in labor at term, having had no prenatal care. After birth, her infant is noted to be small for gestational age with small eyes and a thin upper lip. The infant also is microcephalic. Based on her infant's physical findings, this woman should be questioned about her use of which substance during pregnancy? Select one: a. Cocaine b. Alcohol c. Heroin d. Marijuana

b. Alcohol

During the assessment of the newborn at 3 hours of age, the perinatal nurse documents the presence on the infant's scalp of a unilateral, well-defined mass which does not cross the suture lines. The mother's chart indicates a prolonged labor with use of a vacuum extractor. The RN identifies this finding as: Select one: a. Caput succedaneum b. Cephalohematoma c. Molding d. Intraventricular hemorrhage

b. Cephalohematoma. Cephalohematoma is hematoma formation between the periosteum and skull with unilateral swelling. It appears within a few hours of birth and can increase in size over the next few days. It has a well-defined outline and does not cross suture lines

A pregnant client asks the nurse why she should attend childbirth classes. The nurse's response would be based on which of the following information? Select one: a. Attending childbirth class is a good way to make new friends. b. Childbirth classes will help new families develop skills to meet the challenges of childbirth and parenting. c. Attending childbirth classes will help a pregnant woman have a shorter labor. d. Childbirth classes will help a pregnant woman decrease her chance of having a cesarean delivery.

b. Childbirth classes will help new families develop skills to meet the challenges of childbirth and parenting. CorrectThis is the best answer for the purpose of childbirth classes

Breast pain occurs in many women during their perimenopausal years. Which information is (are) a priority for the nurse to share with the client? (Select all that apply). Select one or more: a. Pain is almost always an indication of a solid mass. b. Distinguishing between cyclical and noncyclical pain is important. c. Assess for prior trauma or possible infection d. Breast pain is an early indication of cancer.

b. Distinguishing between cyclical and noncyclical pain is important. Correct c. Assess for prior trauma or possible infection Correct

Which of the following is true regarding dizygotic twins? Select one: a. The fetuses share one chorion. b. Each fetus has their own placenta. c. The fetuses share one amnion. d. Each fetus has their own uterus.

b. Each fetus has their own placenta.

As the tocodynamometer (Toco) is placed on the laboring patient's abdomen, the nurse explains that this monitoring device provides information on which of the following? Select one: a. Fetal heart rate b. Frequency of contractions c. Intensity of contractions d. Progress of labor

b. Frequency of contractions Uterine contractions are measured via a tocodynamometer (Toco) which is an external uterine monitor. The Toco measures the frequency and duration of uterine contractions but cannot measure uterine pressure/intensity. Uterine pressure/intensity can be estimated by palpation during contractions or through the use of an internal monitor such as an intrauterine pressure catheter (IUPC).

ntrauterine resuscitation techniques are generally initiated by nurses in an effort to improve maternal blood flow to the placenta and oxygen delivery to the fetus. These techniques include: Select one or more: a. Increase uterine activity to enhance blood flow to the fetus b. IV fluid bolus to correct hypovolemia and/or hypotension c. Oxygen by nasal cannula to increase saturation d. Reposition mother off her back and onto the right or left side to correct/prevent aortocaval syndrome.

b. IV fluid bolus to correct hypovolemia and/or hypotension d. Reposition mother off her back and onto the right or left side to correct/prevent aortocaval syndrome.

A 26-year-old primigravida has come to the clinic for her regular prenatal visit at 12 weeks. She appears thin and somewhat nervous. She reports that she eats a well-balanced diet, although her weight is 5 pounds less than it was at her last visit. The results of laboratory studies confirm that she has a hyperthyroid condition. Based on the available data, the nurse formulates a plan of care. Which nursing diagnosis is most appropriate for the client at this time? Select one: a. Imbalanced nutrition: more than body requirements b. Imbalanced nutrition: less than body requirements c. Disturbed sleep pattern d. Deficient fluid volume

b. Imbalanced nutrition: less than body requirements

The nurse is evaluating the EFM tracing of the client who is in active labor. Suddenly, the FHR drops from its baseline of 125 down to 80 beats per minute. The mother is repositioned, and the nurse provides oxygen, increased IV fluids, and performs a vaginal examination. The cervix has not changed. Five minutes have passed, and the FHR remains in the 80s. What additional nursing measures should the nurse take next? Select one: a. Insert a Foley catheter. b. Immediately notify the care provider. c. Call for help. d. Start administering Pitocin.

b. Immediately notify the care provider.

Under which circumstance should the nurse immediately alert the pediatric provider? Select one: a. The infant goes into a deep sleep 1 hour after childbirth. b. Infant is dusky and turns cyanotic when crying. c. Acrocyanosis is present 1 hour after childbirth. d. The infant's blood glucose level is 45 mg/dl.

b. Infant is dusky and turns cyanotic when crying.

What is the most dangerous effect on the fetus of a mother who smokes cigarettes while pregnant? Select one: a. Extensive CNS damage b. Intrauterine growth restriction c. Genetic changes and anomalies d. Fetal addiction to the substance inhaled

b. Intrauterine growth restriction

Which hormone remains elevated in the immediate postpartum period of the breastfeeding woman? Select one: a. Human placental lactogen b. Prolactin c. Progesterone d. Estrogen

b. Prolactin

Which of the following findings, seen in pregnant women in the third trimester, would the nurse consider to be within normal limits? Select one: a. Diplopia b. Pyrosis c. Bradycardia d. Oliguria

b. Pyrosis Correct

A woman who is 6 months pregnant has sought medical attention, saying she fell down the stairs. What scenario would cause an emergency department nurse to suspect that the woman has been a victim of IPV? Select one: a. The woman and her partner are having an argument that is loud and hostile. b. The woman has injuries on various parts of her body that are in different stages of healing. c. Examination reveals a fractured arm and fresh bruises. d. She avoids making eye contact and while answering questions.

b. The woman has injuries on various parts of her body that are in different stages of healing. CorrectSee p 107 of textbook

Human immunodeficiency virus (HIV) may be transmitted perinatally or during the postpartum period. Which statement regarding the method of transmission is most accurate? Select one: a. Only in the third trimester from the maternal circulation b. Through the ingestion of breast milk from an infected mother c. Only through the ingestion of amniotic fluid d. From the use of unsterile instruments

b. Through the ingestion of breast milk from an infected mother

What is the rationale for the administration of an oxytocic (e.g., Pitocin, Methergine) after expulsion of the placenta? Select one: a. To facilitate rest and relaxation b. To stimulate uterine contraction c. To relieve pain d. To prevent infection

b. To stimulate uterine contraction

A 26-year-old pregnant woman, gravida 2, para 1-0-0-1, is 28 weeks pregnant when she experiences bright red, painless vaginal bleeding. On her arrival at the hospital, which diagnostic procedure will the client most likely have performed? Select one: a. Amniocentesis for fetal lung maturity b. Transvaginal ultrasound for placental location can confirm this diagnosis. c. Contraction stress test (CST) d. Internal fetal monitoring

b. Transvaginal ultrasound for placental location The symptoms: bright red and painless vaginal bleeding, indicate placental previa. An ultrasound

The perinatal nurse demonstrates for the student nurse the correct technique of postpartum uterine palpation. Support for the lower uterine segment is critical, as without it, there is an increased risk of: Select one: a. Uterine edema b. Uterine inversion c. Incorrect measurement d. Intensifying the patient's level of pain

b. Uterine inversion- During pregnancy there is stretching of the ligaments that hold the uterus in place. Fundal pressure could result in uterine inversion. Supporting the lower uterine segment may prevent uterine inversion during fundal assessment or massage.

The nurse providing care for a high-risk laboring woman is alert for late FHR decelerations. Which clinical finding might be the cause for these late decelerations? Select one: a. Meconium fluid b. Uteroplacental insufficiency c. Umbilical cord compression d. Altered cerebral blood flow

b. Uteroplacental insufficiency

The perinatal nurse understands that the purpose of the surgical "time-out" is to: Select one: a. Confirm that the surgeon is ready to begin b. Verify that it is the correct patient and planned procedure c. Verify that anesthesia is adequate d. Confirm that the neonatal team is in attendance

b. Verify that it is the correct patient and planned procedure Surgical "time-out" is performed by the entire surgical team and the patient prior to the administration of anesthesia. The purpose is to validate correct patient and planned procedure.

Which information regarding the care of antepartum women with cardiac conditions is most important for the nurse to understand? Select one: a. Stress on the heart is greatest in the first trimester and the last 2 weeks before labor. b. Women with class II cardiac disease should avoid heavy exertion and any activity that causes even minor symptoms. c. Women with class III cardiac disease should get 8 to 10 hours of sleep every day and limit housework, shopping, and exercise. d. Women with class I cardiac disease need bed rest through most of the pregnancy and face the possibility of hospitalization near term.

b. Women with class II cardiac disease should avoid heavy exertion and any activity that causes even minor symptoms.

Many first-time parents do not plan on having their parents' help immediately after the newborn arrives. Which statement by the nurse is the most appropriate when counseling new parents regarding the involvement of grandparents? Select one: a. "You should tell your parents to leave you alone." b. "They are getting old. You should let them be involved while they can." c. "Grandparents can help you with parenting skills." d. "Grandparent involvement can be very disruptive to the family."

c. "Grandparents can help you with parenting skills."

An infant at 36 weeks of gestation has increasing respirations (80 to 100 breaths per minute with significant substernal retractions). The infant is given oxygen by continuous nasal positive airway pressure (CPAP). What level of partial pressure of arterial oxygen (PaO2) indicates hypoxia? Select one: a. 73 mmHg b. 89 mmHg c. 45 mmHg d. 67 mmHg

c. 45 mmHg

The nurse completes an initial newborn examination on a baby boy at 90 minutes of age. The baby was born at 40 weeks' gestation with no birth trauma. The nurse's findings include the following parameters: heart rate 136 beats per minute; respiratory rate 64 breaths per minute; temperature 98.2°F (36.8°C); length 49.5 cm; and weight 3500 g. The nurse documents the presence of a heart murmur, absence of bowel sounds, symmetry of ears and eyes, no grunting or nasal flaring, and full range of movement of all extremities. Which assessment would warrant further investigation and require immediate consultation with the baby's health-care provider? Select one: a. Respiratory rate b. Presence of a heart murmur c. Absent bowel sounds d. Weight

c. Absent bowel sounds

A 25-year-old gravida 1 para 1 who had an emergency cesarean birth 3 days ago is scheduled for discharge. As the nurse prepares her for discharge, she begins to cry. The nurse's next action should be what? Select one: a. Point out how lucky she is to have a healthy baby. b. Explain that she is experiencing postpartum blues. c. Allow her time to express her feelings. d. Assess her for pain.

c. Allow her time to express her feelings. The nurse needs to hear from the patient why she is crying before offering a response.

What is the rationale for evaluating the plantar crease within a few hours of birth? Select one: a. Creases will be less prominent after 24 hours. b. Heel sticks may be required. c. As the skin dries, the creases will become more prominent. d. Newborn has to be footprinted.

c. As the skin dries, the creases will become more prominent. This helps the nurse better evaluate for physical maturity

The nurse observes that a first-time mother appears to ignore her newborn. Which strategy should the nurse use to facilitate mother-infant attachment? Select one: a. Demonstrate for the mother different positions for holding her infant while feeding. b. Arrange for the mother to watch a video on parent-infant interaction. c. Show the mother how the infant initiates interaction and attends to her. d. Tell the mother she must pay attention to her infant.

c. Show the mother how the infant initiates interaction and attends to her.

Another common pregnancy-specific condition is pruritic urticarial papules and plaques of pregnancy (PUPPP). A client asks the nurse why she has developed this condition and what can be done. What is the nurse's best response? Select one: a. This common pregnancy-specific condition is associated with a poor fetal outcome. b. PUPPP is associated with decreased maternal weight gain. c. The goal of therapy is to relieve discomfort. d. The rate of hypertension decreases with PUPPP.

c. The goal of therapy is to relieve discomfort.

What is a distinct advantage of external EFM? Select one: a. Once correctly applied by the nurse, the transducer need not be repositioned even when the woman changes positions. b. The ultrasound transducer can accurately measure short-term variability and beat-to-beat changes in the FHR. c. The tocotransducer is especially valuable for measuring uterine activity during the first stage of labor. d. The tocotransducer can measure and record the frequency, regularity, intensity, and approximate duration of uterine contractions.

c. The tocotransducer is especially valuable for measuring uterine activity during the first stage of labor. CorrectCorrect!

A woman in preterm labor at 30 weeks of gestation receives two 12-mg intramuscular (IM) doses of betamethasone. What is the purpose of this pharmacologic intervention? Select one: a. To reduce maternal and fetal tachycardia associated with ritodrine administration b. To suppress uterine contractions c. To stimulate fetal surfactant production d. To maintain adequate maternal respiratory effort and ventilation during magnesium sulfate therapy

c. To stimulate fetal surfactant production. Important as baby may deliver preterm.

Which condition would require prophylaxis to prevent subacute bacterial endocarditis (SBE) both antepartum and intrapartum? Select one: a. Arrhythmias b. Congestive heart disease c. Valvular heart disease d. Postmyocardial infarction

c. Valvular heart disease

A nurse is discussing the storage of breast milk with a mother whose infant is preterm and in the special care nursery. Which statement indicates that the mother requires additional teaching? Select one: a. "I can store my breast milk at room temperature for 4 hours." b. "I can store my breast milk in the refrigerator for 3 to 5 days." c. "I can store my breast milk in the freezer for 3 months." d. "I can store my breast milk in the refrigerator for 3 months."

d. "I can store my breast milk in the refrigerator for 3 months."

A nurse is making a home visit on the twelfth postpartum day to assess a 23-year-old primipara woman and her full-term, healthy baby. Breastfeeding is the method of infant nutrition. The woman tells the nurse that she does not think her milk is good because it looks very watery when she expresses a little before each feeding. The nurse's best response is: Select one: a. "This is normal. You only have to be concerned when your baby does not gain weight." b. "What types of foods are you eating? A lack of protein in the diet can cause watery looking breast milk." c. "How much fluid are you drinking while you are nursing your baby? Too much fluid during the feeding session can dilute the breast milk." d. "This is normal and is referred to as foremilk which is higher in water content. Later in the feeding the fat content increases and the milk becomes richer in appearance."

d. "This is normal and is referred to as foremilk which is higher in water content. Later in the feeding the fat content increases and the milk becomes richer in appearance." There are 3 stages of human milk production: Stage 1 is colostrum, a yellowish fluid present for 2-3 days after birth and rich is protein; Stage 2 is transitional milk and consists of colostrum and milk and is present from day 3-10; Stage 3 is mature milk and consists of foremilk which is produced and stored between feedings and is higher in water content and hind milk which is produced during the feeding session and is higher in fat content.

Which client is most likely to experience strong and uncomfortable afterpains? Select one: a. A woman who experienced oligohydramnios b. A woman whose infant weighed 5 pounds, 3 ounces c. A woman who is bottle-feeding her infant d. A woman who is a gravida 4, para 4-0-0-4

d. A woman who is a gravida 4, para 4-0-0-4

Nurses can help their clients by keeping them informed about the distinctive stages of labor. Which description of the phases of the first stage of labor is accurate? Select one: a. Latent: Mild, regular contractions; no dilation; bloody show; duration of 2 to 4 hours b. Lull: No contractions; dilation stable; duration of 20 to 60 minutes c. Transition: Very strong but irregular contractions; 8- to 10-cm dilation; duration of 1 to 2 hours d. Active: Moderate, regular contractions; 4- to 7-cm dilation; duration of 3 to 6 hours

d. Active: Moderate, regular contractions; 4- to 7-cm dilation; duration of 3 to 6 hours

An infant boy was delivered minutes ago. The nurse is conducting the initial assessment. Part of the assessment includes the Apgar score. When should the Apgar assessment be performed? Select one: a. Only if the newborn is in obvious distress b. Every 15 minutes during the newborn's first hour after birth c. Once by the obstetrician, just after the birth d. At least twice, 1 minute and 5 minutes after birth

d. At least twice, 1 minute and 5 minutes after birth CorrectCorrect!

The nurse is performing an initial assessment of a client in labor. What is the appropriate terminology for the relationship of the fetal body parts to one another? Select one: a. Position b. Lie c. Presentation d. Attitude

d. Attitude Correct

The nurse is developing a plan of care for a African American client who just delivered a newborn. Which cultural variation is most important to include in the care plan? Select one: a. Maternal grandmother participates in the care of the mother and her infant after birth. b. Male infants are typically circumcised. c. Breastfeeding is encouraged immediately after birth. d. Avoid offering tub baths and shampooing after delivery.

d. Avoid offering tub baths and shampooing after delivery.

Which newborn reflex is elicited by stroking the lateral sole of the infant's foot from the heel to the ball of the foot? Select one: a. Tonic Neck b. Stepping c. Plantar grasp d. Babinski

d. Babinski

A mother refused to allow her son to receive the vitamin K injection at birth. Which of the following signs or symptoms might the nurse observe in the baby as a result? Select one: a. Skin color is dusky. b. Vital signs are labile. c. Glucose levels are subnormal. d. Circumcision site oozes blood.

d. Circumcision site oozes blood. Correct- Vitamin K activates coagulation factors which prevent delayed clotting and hemorrhagic disease

As a powerful central nervous system (CNS) stimulant, which of these substances can lead to miscarriage, preterm labor, placental separation (abruption), and stillbirth? Select one: a. Phencyclidine (1-phenylcyclohexylpiperidine; PCP) b. Alcohol c. Heroin d. Cocaine

d. Cocaine

A primigravida is being monitored at the prenatal clinic for preeclampsia. Which finding is of greatest concern to the nurse? Select one: a. Pitting pedal edema at the end of the day b. Blood pressure (BP) increase to 138/86 mm Hg c. Weight gain of 0.5 kg during the past 2 weeks d. Dipstick value of 3+ for protein in her urine

d. Dipstick value of 3+ for protein in her urine

A number of metabolic changes occur throughout pregnancy. Which physiologic adaptation of pregnancy will influence the nurse's plan of care? Select one: a. Women with insulin-dependent diabetes are prone to hyperglycemia during the first trimester because they are consuming more sugar. b. Maternal insulin requirements steadily decline during pregnancy. c. Insulin crosses the placenta to the fetus only in the first trimester, after which the fetus secretes its own. d. During the second and third trimesters, pregnancy exerts a diabetogenic effect that ensures an abundant supply of glucose for the fetus.

d. During the second and third trimesters, pregnancy exerts a diabetogenic effect that ensures an abundant supply of glucose for the fetus.

The perinatal nurse teaches the postpartum woman about the normal process of diuresis that she can expect to occur approximately 6 to 8 hours after birth. A decrease in which of the following hormones is primarily responsible for the diuresis? Select one: a. Prolactin b. Progesterone c. Lactogen d. Estrogen

d. Estrogen Maternal diuresis (elimination of excess tissue fluids) occurs within 12 hours after birth. After childbirth, a decrease in the level of estrogen naturally occurs and contributes to the diuresis.

Where is the point of maximal intensity (PMI) of the FHR located? Select one: a. In a breech position, heard below the mother's umbilicus b. In a vertex position, heard above the mother's umbilicus c. Usually directly over the fetal abdomen d. Heard lower and closer to the midline of the mother's abdomen as the fetus descends and internally rotates

d. Heard lower and closer to the midline of the mother's abdomen as the fetus descends and internally rotates CorrectBest answer- Correct!

A woman in the active phase of the first stage of labor is using a shallow pattern of breathing, which is approximately twice the normal adult breathing rate. She starts to complain about feeling lightheaded and dizzy and states that her fingers are tingling. Which intervention should the nurse immediately initiate? Select one: a. Tell the woman to slow her pace of her breathing. b. Administer oxygen via a mask or nasal cannula. c. Contact the woman's physician. d. Help her breathe into a paper bag.

d. Help her breathe into a paper bag. She is experiencing respiratory alkolosis and breathing into a paper bag enables her to rebreathe carbon dioxide and replace bicarbonate ions.

The nurse is massaging a boggy uterus. The uterus does not respond to the massage. Which medication would the nurse expect would be given first: Select one: a. Methergine b. Epinephrine c. Carboprost (Hemabate) d. Oxytocin or pitocin

d. Oxytocin or pitocin If the cause of the hemorrhage is uterine atony, continual fundal massage with lower uterine segment support is mandatory. While one member of the team massages the fundus, another nurse establishes intravenous access with a large bore needle and administers oxytocic drugs in the following order: oxytocin (Pitocin), followed by methylergonovine (Methergine), and carboprost (Hemabate).

A family is visiting two surviving triplets. The third triplet died 2 days ago. What action indicates that the family has begun to grieve for the dead infant? Select one: a. Asks about the dead triplet's current status b. Refers to the two live infants as twins c. Brings in play clothes for all three infants d. Refers to the dead infant in the past tense

d. Refers to the dead infant in the past tense

The laboratory results for a postpartum woman are as follows: blood type, A; Rh status, positive; rubella non-immune (titer 1:8 or enzyme immunoassay [EIA] 0.8); hematocrit, 30%. How should the nurse best interpret these data? Select one: a. Kleihauer-Betke test should be performed. b. Rh immune globulin is necessary within 72 hours of childbirth. c. Blood transfusion is necessary. d. Rubella vaccine should be administered.

d. Rubella vaccine should be administered.

The nurse is completing a physical examination of the newborn 24 hours after birth. Which component of the evaluation is correct? Select one: a. The parents are excused to reduce their normal anxiety. b. Once often neglected, blood pressure is now routinely checked. c. When the nurse listens to the neonate's heart, the S1 and S2 sounds can be heard; the S1 sound is somewhat higher in pitch and sharper than the S2 sound. d. The nurse can gauge the neonate's maturity level by assessing his or her general appearance.

d. The nurse can gauge the neonate's maturity level by assessing his or her general appearance.

A new father wants to know what medication was put into his infant's eyes and why it is needed. How does the nurse explain the purpose of the erythromycin (Ilotycin) ophthalmic ointment? Select one: a. This ointment prevents the infant's eyelids from sticking together and helps the infant see. b. Erythromycin (Ilotycin) ophthalmic ointment destroys an infectious exudate caused by Staphylococcus that could make the infant blind. c. Erythromycin (Ilotycin) prevents potentially harmful exudate from invading the tear ducts of the infant's eyes, leading to dry eyes. d. This ophthalmic ointment prevents gonorrheal infection of the infant's eyes, potentially acquired from the birth canal

d. This ophthalmic ointment prevents gonorrheal infection of the infant's eyes, potentially acquired from the birth canal

What is the primary difference between the labor of a nullipara and that of a multipara? Select one: a. Sequence of labor mechanisms b. Level of pain experienced c. Amount of cervical dilation d. Total duration of labor

d. Total duration of labor Correct

Karen, a 26-year-old woman and 12 weeks pregnant, has come for her prenatal visit. She asks about avoiding uncooked meat. She also got a new cat and she has heard that she "should not touch the cat during pregnancy." Are these true? The clinic nurse's best response is: Select one: a. It is best if someone other than you changes the cat's litter pan during pregnancy so that you have no risk of toxoplasmosis during pregnancy. b. It is important to have someone else change the litter pan during pregnancy and also avoid consuming raw vegetables. c. Have you had any "flu-like" symptoms since you got your cat? If so, you may have already had toxoplasmosis and there is nothing to worry about. d. Toxoplasmosis is a concern during pregnancy, so it is important to have someone else change the cat's litter pan and also to avoid consuming uncooked meat.

d. Toxoplasmosis is a concern during pregnancy, so it is important to have someone else change the cat's litter pan and also to avoid consuming uncooked meat. Correct Best answer.

A pregnant woman's diet consists almost entirely of whole grain breads and cereals, fruits, and vegetables. Which dietary requirement is the nurse most concerned about? Select one: a. Protein b. Calcium c. Folic Acid d. Vitamin B12

d. Vitamin B12 CorrectCorrect!

Which client is at greatest risk for early PPH? Select one: a. Primigravida in spontaneous labor with preterm twins b. Primiparous woman (G 2, P 1-0-0-1) being prepared for an emergency cesarean birth for fetal distress c. Multiparous woman (G 3, P 2-0-0-2) with an 8-hour labor d. Woman with severe preeclampsia on magnesium sulfate whose labor is being induced

d. Woman with severe preeclampsia on magnesium sulfate whose labor is being induced

The perinatal nurse explains to the student nurse that in the fetal circulation, the lowest level of oxygen concentration is found in _________________. Select one: a. the foramen ovale b. the descending aorta. c. the umbilical vein. d. the umbilical arteries.

d. the umbilical arteries. Correct


Ensembles d'études connexes

Essential Cell Biology Chapter 14: Energy Generation in Mitochondria and Chloroplasts (Question)

View Set

Chapter 6 Egoism, Self-Interest & Altruism

View Set

Bones of the Appendicular Skeleton

View Set

AP Psychology Chapter 4 Study Guide

View Set

Chapter 14: The Protestant Reformation (14.3)

View Set

Anatomy and Physiology chapter 1

View Set